0% found this document useful (0 votes)
104 views134 pages

Rankers Advanced Tudy Material: Rankers Varanasi Office - Pandeypur Chauraha - Call 8601467389

The document discusses kinetic energy, work done by variable forces, and power. It defines kinetic energy as the internal capacity of an object to do work due to its motion. Kinetic energy depends on the mass and speed of an object. It also defines work done by a variable force and provides a formula to calculate the total work done. Finally, it defines power as the rate at which work is done over time and provides the formula to calculate average and instantaneous power. Examples are also included to demonstrate calculating velocity, work done, and potential energy.

Uploaded by

Vishal Jindal
Copyright
© © All Rights Reserved
We take content rights seriously. If you suspect this is your content, claim it here.
Available Formats
Download as PDF, TXT or read online on Scribd
0% found this document useful (0 votes)
104 views134 pages

Rankers Advanced Tudy Material: Rankers Varanasi Office - Pandeypur Chauraha - Call 8601467389

The document discusses kinetic energy, work done by variable forces, and power. It defines kinetic energy as the internal capacity of an object to do work due to its motion. Kinetic energy depends on the mass and speed of an object. It also defines work done by a variable force and provides a formula to calculate the total work done. Finally, it defines power as the rate at which work is done over time and provides the formula to calculate average and instantaneous power. Examples are also included to demonstrate calculating velocity, work done, and potential energy.

Uploaded by

Vishal Jindal
Copyright
© © All Rights Reserved
We take content rights seriously. If you suspect this is your content, claim it here.
Available Formats
Download as PDF, TXT or read online on Scribd
You are on page 1/ 134

RASM

Rankers Advanced Study Material

Rankers Varanasi office | Pandeypur Chauraha – Call 8601467389


WORK , ENERGY AND POWER

KINETIC ENERGY :
Definition : Kinetic energy is the internal capacity of doing work of the object by virtue of its
motion. Kinetic energy is a scalar property that is associated with state of motion of an object.
An aero-plane in straight and level flight has kinetic energy of translation and a rotating wheel
on a machine has kinetic energy of rotation. If a particle of mass m is moving with speed ‘v’
1
much less than the speed of the light than the kinetic energy ‘K’ is given by K = mv 2
2

Important Points for K.E.



1. As mass m and v2 ( v.v ) are always positive, kinetic energy is always positive scalar i.e,
kinetic energy can never be negative.

2. The kinetic energy depends on the frame of reference,

p2 2mK ; P = linear momentum


K= and P =
2m

The speed v may be acquired by the body in any manner. The kinetic energy of a group of
particles or bodies is the sum of the kinetic energies of the individual particles. Consider
  
a system consisting of n particles of masses m1, m2, ......, mn. Let v1 , v 2 , ..... , v n be their
respective velocities. Then, the total kinetic energy Ek of the system is given by

1 1 1
Ek = m1v12 + m2v22 + ......... + mnvn2
2 2 2

If m is measured in gram and v in cm s–1, then the kinetic energy is measured in erg. If m
is measured in kilogram and v in m s–1, then the kinetic energy is measured in joule. It may
be noted that the units of kinetic energy are the same as those of work. Infect, this is true
of all forms of energy since they are inter-convertible.

Typical kinetic energies (K) :

S.No. Object Mass (kg) Speed (m s–1) K(J)


–26
1 Air molecule ≈10 500 ≈ 10–21
2 Rain drop at terminal speed 3.5 × 10–5 9 1.4 × 10–3
3 Stone dropped from 10 m 1 14 10 2
4 Bullet 5 × 10–5 200 10 3
5 Running athlete 70 10 3.5 × 103
6 Car 2000 25 6.3 × 105

WORK DONE BY A VARIABLE FORCE :

Rankers Varanasi office | Pandeypur Chauraha – Call 8601467389


When the magnitude and direction of a force vary in three dimensions, it can be expressed as
a function of the position. For a variable force work is calculated for infinitely small
displacement and for this displacement force is assumed to be constant
 
dW = F.ds

The total work done will be sum of infinitely small work

B  B

WA → B = ∫ F ⋅ ds = ∫ (Fcos θ)ds
A A

In terms of rectangular components,

 
F = Fx î + Fy ĵ + Fz k̂ ⇒ ds = dx î + dy ĵ + dz k̂

x y z
B B B
WA →B = ∫ Fx dx +
x
∫ Fy dy +
y
∫ F dz
z
z
A A A

Example 1. The velocity block A of the system shown in figure is VA at any instant. Calculate velocity
of bock B at that instant.

Solution : Work done by internal tension is zero.

16T

8T
8T

4T
4T

2T
2T

T
T
XB A
<
<
<
<

B XA

Rankers Varanasi office | Pandeypur Chauraha – Call 8601467389


∴ 15 T × XB – T × XA = 0

XA = 15 XB

∴ VA = 15 VB

Power :

Power is defined as the time rate of doing work.

When the time taken to complete a given amount of work is important, we measure the
power of the agent of doing work.

The average power ( P or pav) delivered by an agent is given by

W
P or pav = where W is the amount of work done in time t.
t

Power is the ratio of two scalars- work and time. So, power is a scalar quantity. If time taken
to complete a given amount of work is more, then power is less. For a short duration dt, if P
is the power delivered during this duration, then
  
F ⋅ dS  dS  
P= = F⋅ = F.v
dt dt

This is instantaneous power. It may be +ve, –ve or zero.

By definition of dot product,

P = Fv cos θ
 
where θ is the smaller angle between F and v .

This P is called as instantaneous power if dt is very small.

Example 2

A chain of mass M is kept on a hemisphere as shown. Find potential energy of the


chain.

Solution :
M
θ
Rd

R

Rsinθ
θ
U=0

Rankers Varanasi office | Pandeypur Chauraha – Call 8601467389


arc
We know that =θ
Radius

∴ elemental length = Rdθ

M 2M
∴ dm = dθ = dθ
π/2 π

 2M 
Now dU = dmgh =  dθ  (g)(R sinθ)
 π 

U π/2
2M
∴ ∫ dU =
0 π
Rg ∫ sin θ dθ
0

2MgR
( – cos θ )0
π/2
U=
π

 2R   2R 
⇒ U = Mg   [Note that   is the height of COM]
 π   π 

S.No. Conservative forces Non-Conservative forces

1 Work done does not depend upon path Work done depends on path.

2 Work done in round trip is zero. Work done in a round trip is not zero.

Forces are velocity-dependent and


3 Central in nature.
retarding in nature.

When only a conservative force acts within a


systrem, the kinetic enrgy and potential Work done against a non-
4 energy can change. However their sum, the conservative force may be dissipated
mechanical energy of the system, does not as heat energy.
change.

Work done in not completely


5 Work done is completely recoverable.
recoverable.

POTENTIAL ENERGY AND CONSERVATIVE FORCE :

FS = – ∂ U/ ∂ s,

i.e., the projection of the field force, the vector F, at a given point in the direction of the
displacement dr equals the derivative of the potential energy U with respect to a given

Rankers Varanasi office | Pandeypur Chauraha – Call 8601467389


direction, taken with the opposite sign. The designation of a partial derivative ∂ / ∂ s
emphasizes the fact of deriving with respect to a definite direction.

So, having reversed the sign of the partial derivatives of the function U with respect to x, y, z,
we obtain the projection Fx, Fy and Fz of the vector F on the unit vectors i, j and k. Hence, one
can readily find the vector itself : F = Fxi + Fyj + Fzk, or

When conservative force does positive work then PE decreases

dU = – dw

dU = – F.ds

dU = – (Fx î + Fy ĵ + Fz k̂ ) . (dx î + dy ĵ + dz k̂ )

dU = – Fxdx – Fydy – Fzdz

if y & z are constants then dy = 0 dz = 0

dU = –Fxdx

dU
∴ Fx = – if y & z are constant
dx

−∂U
≡ Fx =
∂x

−∂u −∂u
Similarly Fy = ; Fz =
∂y ∂z

 ∂U ∂U ∂U 
F=–  i+ j+ k .
 ∂x ∂y ∂z 

The quantity in parentheses is referred to as the scalar gradient of the function U and is
denoted by grad U or ∇U. We shall use the second, more convenient, designation where ∇
(“nabla”) signifies the symbolic vector or operator

∂ ∂ ∂
∇= i + j +k
∂x ∂y ∂z

Potential Energy curve :

 A graph plotted between the PE a particle and its displacement from the centre of force
field is called PE curve.

 Using graph, we can predict the rate of motion of a particle at various positions.

Rankers Varanasi office | Pandeypur Chauraha – Call 8601467389


dU
 Force on the particle is F(x) = –
dx

Case-I : On increasing x, if U increases, force is in (–) ve x direction i.e. attraction force.

Case-II : On increasing x, if U decreases, force is in (+) ve x-direction i.e. repulsion


force.

EQUILIBRIUM OF A PARTICLE

Different positions of a particle :

Position of equilibrium : If net force acting on a body is zero, it is said to be in equilibrium. For
dU
equilibrium = 0. Points P, Q & R are the states of equilibrium positions.
dx

Types of equilibrium :

 Stable equilibrium : When a particle is displaced slightly from a position and a force acting
on it brings it back to the initial position, it is said to be in stable equilibrium position.

dU d2U
Necessary conditions : – = 0, and = +ve
dx dx 2

 Unstable Equilibrium : When a particle is displaced slightly from a position and force
acting on it tries to displace the particle further away from the equilibrium position, it is
said to be in unstable equilibrium.

dU d2U
Condition : – = 0 potential energy is maximum i.e. = 2 = – ve
dx dx

 Neutral equilibrium : In the neutral equilibrium potential energy is constant. When a


particle is displaced from its position it does not experience any force acting on it and
continues to be in equilibrium in the displaced position. This is said to be neutral
equilibrium.

A particle is in equilibrium if the acceleration of the particle is zero. As acceleration is


frame dependent quantity therefore equilibrium depends on motion of observer also

MECHANICAL ENERGY :

Definition: Mechanical energy ‘E’ of an object or a system is defined as the sum of kinetic
energy ‘K’ and potential energy ‘U’, i.e., E = K + U

Rankers Varanasi office | Pandeypur Chauraha – Call 8601467389


Important Points for M.E.:

1. It is a scalar quantity having dimensions [ML2T-2] and SI units joule.

2. It depends on frame of reference.


3. A body can have mechanical energy without having either kinetic energy or potential
energy. However, if both kinetic and potential energies are zero, mechanical energy will
be zero. The converse may or may not be true, i.e., if E = 0 either both PE and KE are zero
or PE may be negative and KE may be positive such that KE + PE = 0.

4. As mechanical energy E = K + U, i.e., E - U = K. Now as K is always positive, E - U ≥ 0, i.e.,


for existence of a particle in the field, E ≥ U.
5. As mechanical energy E = K + U and K is always positive, so, if ‘U’ is positive ‘E’ will be positive.
However, if potential energy U is negative, ‘E’ will be positive if K > |U| and E will be negative
if K < |U|
i.e., mechanical energy of a body or system can be negative, and negative mechanical
energy means that potential energy is negative and in magnitude it is more than kinetic
energy. Such a state is called bound state, e.g., electron in an atom or a satellite moving
around a planet are in bound state.

Example 3 A rigid body of mass m is held at a height H on two smooth wedges of mass M each of
which are themselves at rest on a horizontal frictionless floor. On releasing the body
it moves down pushing aside the wedges. The velocity of recede of the wedges from
each other when rigid body is at a height h from the ground is

2mg(H − h) 2mg(H − h) 8mg(H − h) 8mg(H − h)


(A) (B) (C) (D)
m + 2M 2m + M m + 2M 2m + M

Solution :

Rankers Varanasi office | Pandeypur Chauraha – Call 8601467389


Let speed of the wedge and the rigid body be V and υ respectively.

Then applying wedge constraint we get

V cos 45º = υ cos 45º

∴ V=υ ...(i)

Using energy conservation,

1  1
mg(H – h) = 2  MV 2  + mυ2 ...(ii)
2  2

From equation (i) and (ii)

2mg(H − h)
V=
m + 2M

8mg(H − h)
∴ The velocity of recede of wedges from each other = 2 × V =
m + 2M

So, answer is (C)

Alter : Length of rod = 


x+y=
2

dx dy
+ =0
dt dt

velocity of block = velocity of rod

decrease in potential energy = increase in kinetic energy

1 1 1
mg (H – h) = mV2 + MV2 + MV2
2 2 2

2mg(H − h)
∴ V=
2M + m

8mg(H − h)
∴ 2V =
2M + m

Rankers Varanasi office | Pandeypur Chauraha – Call 8601467389


ADVANCED LEVEL PROBLEM
1. A block of mass ' m ' is pushed against a spring of spring constant
' k ' fixed at one end to a wall. The block can slide on a frictionless
table as shown in the figure. The natural length of the spring is
L0 and it is compressed to one-fourth of natural length and the
block is released. Find its velocity as a function of its distance (x)
from the wall and maximum velocity of the block. The block is
not attached to the spring.

2. A block of mass m rests on a rough horizontal plane having


coefficient of kinetic friction µk and coefficient of static friction
µs. The spring is in its natural length, when a constant force of
5 µk mg
magnitude P = starts acting on the block.
4

The spring force F is a function of extension x as F = kx3. (Where k is spring constant)

(a) Comment on the relation between µs and µk for the motion to start.

(b) Find the maximum extension in the spring (Assume the force P is sufficient to make the block
move).

3. A particle of mass m = 1 kg lying on x-axis experiences a force given by



law F = x (3x – 2) î Newton,

where x is the x-coordinate of the particle in meters.

(a) Locate the points on x-axis where the particle is in equilibrium.

(b) Draw the graph of variation of force F (y-axis) with x-coordinate of the particle (x-axis).
Hence or otherwise indicate at which positions the particle is in stable or unstable
equilibrium.

(c) What is the minimum speed to be imparted to the particle placed at x = 4 meters such that
it reaches the origin.

4. A ring of mass ' m ' can slide along a fixed rough vertical rod as shown
4m g
in fig. The ring is connected by a spring of spring constant k =
R
where 2 R is the natural length of spring. The other end of spring is fixed
to the ground at point A at a horizontal distance of 2 R from the base of
the rod. If the ring is released from a height of 3R/2 & it reaches the
ground with a speed 3 gR , find co-efficient of friction between the rod
& ring.

Rankers Varanasi office | Pandeypur Chauraha – Call 8601467389


5. A block of mass m1 is kept over another block of mass m2 and the
system rests on a horizontal surface (as shown in figure). A constant
horizontal force 2F acting on the lower block produces an acceleration
F
in the system, the two blocks always move together.
(m1 + m2 )

(a) Find the coefficient of kinetic friction between the bigger block and the horizontal surface.

(b) Find the frictional force acting on the smaller block.

(c) Find the work done by the force of friction on the smaller block by the bigger block during
a displacement x of the system.

6. A box having mass 400 kg is to be slowly slide through 10 m on a horizontal straight track
having friction coefficient 0.2 with the box.

(a) Find the work done by the person pulling the box with a rope at an angle θ with the
horizontal.

(b) Find the work when the person has chosen a value of θ which ensures him the minimum
magnitude of the force. [g = 10 m/s2]

7. A small bead ‘B’ of mass m is free to slide on a fixed smooth vertical wire, as
indicated in the diagram. One end of a light elastic string, of unstreched length a
and force constant 2mg/a is attached to B. The string passes through a smooth
fixed ring R and the other end of the string is attached to the fixed point A, AR being
horizontal. The point O on the wire is at same horizontal level as R and
AR = RO = a.

(i) In the equilibrium position, find OB.

(ii) The bead B is raised to a point C of the wire above O, where OC = a and is released from
rest. Find the speed of the bead as it passes O and find the greatest depth below O of the
bead in the subsequent motion.

8. A particle of mass m approaches a region of force starting from r = + ∞. The potential energy
function in terms of distance r from the origin is given by,

K
U (r) = (3 a2 − r2) for, 0 ≤ r ≤ a
2a3

= K/r for, r ≥ a

where K > 0 (positive constant)

(a) Derive the force F(r) and determine whether it is repulsive or attractive.

(b) With what velocity should the particle start at r = ∞ to cross over to other side of the origin.

2K
(c) If the velocity of the particle at r = ∞ is towards the origin describe the motion.
am

Rankers Varanasi office | Pandeypur Chauraha – Call 8601467389


9. A uniform string of mass ‘M’ and length 2a, is placed symmetrically over a smooth and small
pulley and has particles of masses ‘m’ and ‘m′’ attached to its ends; show that when the string

runs off the peg its velocity is √. MM++2(m − m′) 


m + m′
ag Assume that m > m′.

10. A single conservative force F(x) acts on a 1.0 kg particle that moves along the x-axis. The
potential energy U(x) is given by : U(x) = 20 + (x – 2)2 where x is in meters. At x = 5.0 m the
particle has a kinetic energy of 20 J.

(i) What is the mechanical energy of the system ?

(ii) Make a plot of U (x) as a function of x for – 10 m < x < 10m, and on the same graph draw
the line that represents the mechanical energy of the system. Use part (ii) to determine

(iii) The least value of x and

(iv) The greatest value of x between which the particle can move.

(v) The maximum kinetic energy of the particle and

(vi) The value of x at which it occurs.

(vii) Determine the equation for F (x) as a function of x.

(viii) For what value of x does F(x) = 0 ?

11. A 1.2 kg collar C may slide without friction along a fixed smooth
horizontal rod. It is attached to three springs each of constant
K = 400 N/m and 150 mm undeformed length. Knowing that the collar
is released from rest in the position shown. Determine the maximum
velocity it will reach in its motion. [Here A, O, B are fixed points.]

12. A block of mass 4 kg is moved from rest on a smooth inclined plane of inclination 53° by
applying a constant force of 40 N parallel to the incline. The force acts for two seconds. (a)
Show that the work done by the applied force is not less than 160 J. (b) Find the work done by
the force of gravity in that two seconds if the work done by the applied force is 160 J. (c) Find
the kinetic energy of the block at the instant the force ceases to act in case (b). [Take g = 10
m/s2]

13. There is a vertically suspended spring, mass system. When block of mass 10 kg is suspended
from lower end of the spring, it is stretched by 20 cm under the load of block at equilibrium
position. When an upward speed of 4 m/s is imparted to the block by giving a sharp impulse
from below, how much high will it rise from equilibrium position.

14. A certain spring is found not to obey Hooke’s law, it exerts a restoring force F(x) = –αx – βx2 if
it is stretched or compressed, where α = 48 N/m and β = 24 N/m2. The mass of the spring is
negligible. An object with mass 1 kg on a frictionless, horizontal surface is attached to the
spring, pulled a distance 1m to the right to stretch the spring and released. The speed of the
object when it is 0.5m to the right of the x = 0 equilibrium position is

Rankers Varanasi office | Pandeypur Chauraha – Call 8601467389


15. Wind entering in a wind mill with a velocity of 20 m/sec facing area of the windmill is 10m2 and
density of air is 1.2 kg/m3. If wind energy is converted into electrical energy with 33.3%
efficiency, then find electrical power produced by the wind mill in kw.

16. An engine can pull 4 coaches at a maximum speed of 20 m/s. Mass of the engine is twice the
mass of every coach. Assuming resistive forces to be proportional to the weight, approximate
maximum speeds of the engine when it pulls 12 and 6 coaches are (power of engine remains
constant) :

17. A pump motor is used to deliver water at a certain rate from a given pipe. To obtain “n” times
water from the same pipe in the same time, the factor by which the power of the motor should be
increased is: [REE 1998]

18. A chain of mass M and length  is held vertically such that its bottom end just touches the
surface of a horizontal table. The chain is released from rest. Assume that the portion of chain
on the table does not form a heap. The momentum of the portion of the chain above the table

after the top end of the chain falls down by a distance .
8

V
19. The figure shows a hollow cube of side 'a' of volume V. There is a small chamber of volume
4
in the cube as shown. This chamber is completely filled by m kg of water. Water leaks through
a hole H and spreads in the whole cube. Then the work done by gravity in this process assuming
that the complete water finally lies at the bottom of the cube is :

20. From what minimum height ‘h’ in metre must the system be released when spring is in its natural
length as shown in the figure. So that after perfectly inelastic collision. (e = 0), of block B, with
ground, B may be lifted off ground. (Take k = 40 N/m, g = 10 m/s2, mA = 2kg, mB = 4kg)

Rankers Varanasi office | Pandeypur Chauraha – Call 8601467389


ANSWER KEY

k   3 L0  
2
2 3 L0 k
1. v=   − (L0 − x )  when x< L0 ; vmax = when x ≥ L0
m  4   4 m

1/ 3
 µ mg 
2. (a) 5µk > 4µs ; (b) x =  K 
 K 

2
3. (a) x = 0 and x = m (b) The particle is in stable equilibrium
3

2 2600
at x = 0 metre and unstable equilibrium at x = metre (c) v = m/s
3 27

1 F m1F m1Fx
4. 5. (a) (b) (c)
8(3 − 4  n 2) (m1 + m2 )g (m1 + m2 ) (m1 + m2 )

40000
6. (a) J (b) 7692.31 J 7690 J 7. (i) OB = a/2 (ii) v = 4ag , d
5 + tan θ
= 2a

3k
8. (a) repulsive (b) v > (c) stops at r = a & then reaches to r = ∞ .
am

U(x)
164

84
49
ME
20
10. (i) 49 J (ii) (iii) − 29 + 2 ≈ –3.38 m
x
–x x=2 10
7.38
–3.38

–10

(iv) − 29 + 2 ≈ 7.38 m (v) 29 J (vi) x = 2m (vii) F = 2 (2 – x)

(viii) x = 2

1/ 2
11.
 15

{ 2 2 
 2 ( 5 − 1) + ( 2 − 1) 

} m/s = 3.189 m/s 12. (b) – 128 J (c) 32 J

13. 0.56 m = 56 cm 14. 5 2 m/s 15. 16

Rankers Varanasi office | Pandeypur Chauraha – Call 8601467389


7
16. 8.5 m/s and 15 m/s respectively 17. n3 18. M g
16

5
19. mga 20. 2
8

SOLUTION

2
1  3 L0  1 1
1. Solution: k  = mv2 + k (L0 − x)2 when tc x < L0
2  4  2 2

k   3 L 2 2
⇒ v=  0
 − ( L 0 − x ) 
m   4  
2
1  3L0  1 3L0 k
when tc x ≥ L0 K  = mv2 ⇒ v=
2  4  2 4 m

3 L0 k
which is also the maximum speed of the block. Thus, vmax =
4 m
2. Sol. (a) For motion to start

5 µk mg
> µsmg or 5µk > 4µs
4

(b)

Rankers Varanasi office | Pandeypur Chauraha – Call 8601467389


At the final position of the block extension in spring is maximum and the speed of the block is
v = 0. Hence the net work done in taking the block from initial to final position

∆W = work done by P + work done by spring force F + work done by friction = ∆K = 0

x
5 µk mg kx 4
∫ Kx . dx – µkmgx =
3
= Px – .x – –µkmgx = 0
0
4 4

1/ 3
 µ mg 
solving we get x =  K 
 K 

2
3. Solution (a) x = 0 and x = m
3

(b)

2
The particle is in stable equilibrium at x = 0 metre and unstable equilibrium at x =
3
metre

x=0

2600
(c) v= m/s
27
4. Sol. Using work energy thoerem,
2
1
( )
2
3R 1  4mg  R 
W f + mg + = m 3gR
2 2  k  
 2  2

1
⇒ Wf = − mgR
2

⇒ f = µ N (as kinetic friction)

Wf = ∫ f dx = ∫ µ Fs cos θ d x ( x = 2 R tan θ ; dx = 2 R sec2 θ dθ)

Fs = k 2 R(sec θ − 1)

Rankers Varanasi office | Pandeypur Chauraha – Call 8601467389


Wf = µ ∫ k 2 R (sec θ − 1) cos θ × 2 R sec2 θ dθ

0
Wf = 4 R2 µk ∫ (sec2 θ − sec θ) d θ θ0 = tan −1 (3/4)
θ0

mgR
[ tan θ −  n (sec θ + tan θ) ]θ
0
= 4 R2 µk = –4R2µk [tan θ0 − ln (sec θ0 + tan θ0)] = −
0 2

3 5 3 mg R
⇒ 4 R2 µk  4 − n  4 + 4   = R2 µk [3 − 4 ln 2] =
2
  

mg 1
µ= = Ans.
2 Rk (3 − 4  n2) 8 (3 − 4  n 2)

F F
5. Solution : a= f1 = m1a = m1 Ans. of (b)
(m1 + m2 ) (m1 + m2 )

2F – f1 – f2 = m2a

– f2 = – 2F + f1 + m2a = m1a + m2a – 2F

F
– f2 = (m2 + m1) – 2F = F – 2F = – F ⇒ f2 = – F
(m1 + m2 )

2F – µK (m2 + m1)g = (m2 + m1)a

F F
2F – (m2 + m1) = µK (m2 + m1)g ⇒ = µK Ans. of (a)
(m1 + m2 ) (m1 + m2 )g

W = work done by friction force on smaller block

m1F
= f1x = x Ans of (c)
(m2 + m1 )

Rankers Varanasi office | Pandeypur Chauraha – Call 8601467389


6. Sol.

mg = N + F sinθ .......(1)

µN = F cosθ .......(2)

µ mg = F cos θ + µF sin θ

µmg
F= .......(3)
cos θ + µ sin θ

μ mgcos θ (10) 40000


WF = = Ans.
cos θ + μsin θ 5 + tan θ

2
F is min. if D = cos θ + µ sin θ is maximum and its maximum value is 1+ µ

µmg 1 + µ 2 µmg
Fmin. = =
1 + µ2 1 + µ2

µmg 10
WFmin =
1 + µ2 1+ µ 2

µ = 0.2, mg = 4000 Nt

(0.2)(4000)10
WFmin =
( 1 + (0.2)2 )2

400 × 20 8000
= = = 7692.307 J Ans.
( 1 + 0.04 ) 2
1.04

7. Sol. (i)

Rankers Varanasi office | Pandeypur Chauraha – Call 8601467389


mg = T cos θ

2mg  2  x 
mg = a + x2 + a − a ×  
a   2 2
 a +x 

a
∴ x=
2

(ii)

1 1 1 2
( )
2
K 2a + mga = K (2a – a)2 + mv
2 2 2
1  2mg  1 2
  (2a2 – a2) + mga = mv
2 a  2

⇒ 4ga = v

(iii)

1
( ) 1
( )
2 2
K 2a + mg a = K a2 + y 2 – mg y
2 2
1 1 2 2
K2a2 + mg a = K(a + y ) – mgy
2 2
1 2mg 2 1 2mg 2 1 2mg 2
2a + mga = a + y – mg y
2 a 2 a 2 a

mg y 2
3 mg a – mg a = – mg y
a

mg y 2
2 mga = – mg y
a

Rankers Varanasi office | Pandeypur Chauraha – Call 8601467389


2a2 = y2 – ay

⇒ y2 – ay – 2a2 = 0

y2 + ay – 2ay – 2a2

⇒ y (a + y) = 2a (y + a)

⇒ y = 2a

du K K
8. Sol. = 3
(–2r) ⇒ F(r) = 3 r
dr 2a a
 K 
F(r) =  3 r  rˆ for 0 ≤ r ≤ a
a 

 K
F(r) = 2 r̂ for r ≥ a
r
∴K>0

∴ force is +ve

∴ force is repulsive Ans.

1 K3a 2
(b) m u2 + 0 = 0 +
2 2a3

3K 3k
u2 = ⇒ u= Ans.
ma am

1 2k k
(c) m + 0 = 0 + P.E. ⇒ P.E = ⇒ r=a Ans
2 am a

9. Solution

a 1
– g [ma + m´a + M ] = – g [ m2a + m´0 + Ma] + (M + m + m´)v2
2 2

Rankers Varanasi office | Pandeypur Chauraha – Call 8601467389


Ma
2g[2ma − ma + Ma − − m´a]
2 =v
M + m + m'

M + 2(m – m´)
v= ag Ans.
M + m + m´
10. Sol. U (x) = 20 + (x – 2)2

du
= 2(x – 2)
dx
– F = 2(x – 2)

F = – 2(x – 2)

m (x – 2) = – 2 (x – 2)

Let x = x – 2

mx = – 2 x

1x =–2x

x=–2x Simple Harmonic Motion

Mean position is x = x – 2 = 0 ⇒ x=2

W2 = 2 ,

1 2
Kinetic energy = mv
2
1
= (1) (ω2) (A2 – x2) = x – 2, x = 5 – 2 = 3
2
1
20 = (1) (2) {A2 – 32}
2

20 = A2 – 9 ⇒ A2 = 29 ⇒ A= 29
Aliter :

for mean position,

dU
F=– = – 2(x – 2) = 0 ⇒ x=2
dx
At x = 5

Rankers Varanasi office | Pandeypur Chauraha – Call 8601467389


K.E. = 20 J

U(x = 5) = 20 + (5 – 2)2 = 29 J

Total energy T.E. = 20 + 29 = 49 J

At amplitude

U(x)max = 49 J = 20 + (x – 2)2

29 J = (x– 2)2

x=2± 29

x=2+ 29 , 2 – 29

xmin = 2 – 29 = –3.38

xmax = 2 + 29 = 7.38
K.E.max when U(x) is minimum at x = 2

U(x)min = 20 J

KEmax = 29 J

11. Sol. Velocity will be maximum when a = 0

For a = 0, F = 0

This situation occurs for ve following arrangement of springs.

150 150

Natural length is c = 150

Now , Ui + Ki = Uf + Kf

1 1
Ui = K{ 5 c – c}2 + K{ 2 c – c}2
2 2
Ki = 0

1
Uf = 2. K{ 2 c – c}2
2

Rankers Varanasi office | Pandeypur Chauraha – Call 8601467389


1 1
∴ K{ 5 c – c}2 + K{ 2 c – c}2
2 2
1 2 1
= mv + 2. K{ 2 c – c}2
2 2
Solving the equation & putting the

we have
1/ 2
 15
v= 
2
{ 
( 5 − 1)2 + ( 2 − 1)2 

} m/s = 3.189 ms–1

19. Sol. (C) Let h be the height of water surface, finally

a a a
a2h = a. . ; h=
2 2 4

a a 3a 5a
∴ C.M. gets lowered by a –  + = a – =
4 8 8 8

5a
∴ Work done by gravity = mg
8

a a 3a 5a
∴ a– 4 + 8 =a– 8 = 8
 
5a
∴ = mg
8

JEE (ADVANCED) (PREVIOUS YEARS)

Rankers Varanasi office | Pandeypur Chauraha – Call 8601467389


1. STATEMENT - 1 : [JEE 2007' 3/184]

A block of mass m starts moving on a rough horizonntal surface with a velocity v. It stops due
to friction between the block and the surface after moving through a certain distance. The
surface is now tilted to an angle of 30º with the horizontal and the same block is made to go
up on the surface with the same initial velocity v. The decrease in the mechanical energy in
the second situation is smaller than that in the first situation.

Because

STATEMENT - 2

The coefficient of friction between the block and the surface decreases with the increase in
the angle of inclination.

(A) Statement-1 is True, Statement-2 is True; Statement-2 is a correct explanation for


Statement-1

(B) Statement-1 is True, Statement-2 is True; Statement-2 is NOT a correct explanation for
Statement-1

(C*) Statement-1 is True, Statement-2 is False

(D) Statement-1 is False, Statement-2 is True.

Sol. In horizontal plane Kinetic Energy of the block is completely converted into heat due to
Friction but in the case of inclined plane some part of this Kinetic Energy is also convert into
gravitational Potential Energy. So decrease in the mechanical energy in second situation is
smaller than that in the first situation. So statement-1 is correct.

Cofficient of Friction does not depends on normal reaction, In ΙΙ case normal reaction changes
with inclination but not cofficient of friction so this statement is wrong.

Sol.

2.# A block (B) is attached to two unstretched springs S1 and S2 with spring constants k and 4 k,
respectively (see figure Ι). The other ends are attached to identical supports M1 and M2 not
attached to the walls. The springs and supports have negligible mass. There is no friction
anywhere. The block B is displaced towards wall 1 by a small distance x (figure ΙΙ) and released.
The block returns and moves a maximum distance y towards wall 2. Displacements x and y are

Rankers Varanasi office | Pandeypur Chauraha – Call 8601467389


y
measured with respect to the equilibrium position of the block B. The ratio is
x
[JEE 2008, 3/163]

1 1
(A) 4 (B) 2 (C*) (D)
2 4

Sol.

As springs and supports (m1 and m2) are having negligible mass. Whenever springs pull the
massless supports, springs will be in natural length. At maximum compression, velocity of B
will be zero.

And by energy conservation

1 1 y 1
(4K) y2 = Kx2 = Ans. (C)
2 2 x 2

3. A light inextensible string that goes over a smooth fixed pulley as shown in the figure connects
two blocks of masses 0.36 kg and 0.72 kg. Taking g = 10 m/s2, find the work done (in joules)
by the string on the block of mass 0.36 kg during the first second after the system is released
from rest.

Rankers Varanasi office | Pandeypur Chauraha – Call 8601467389


[ JEE 2009, 4/160, –1]

Ans. 8

2m1m2 2 × 0.72 × 0.36


Sol. T= g = × 10
m1 + m2 0.72 + 0.36

T = 4.8 N

m1 − m2 g
a= g =
m1 + m2 3

1 2 1 g 10
s= at =  3  (1) = 6
2
2 2  
Work done by T = (T) (S)

10
= (4.8) × =8J Ans.
6

 
x ˆi + y ˆj  (K
4. The work done on a particle of mass m by a force, K 
(
 x2 + y2
) (x ) 
3/2 3/2
2
+ y2
 
being a constant of appropriate dimensions), when the particle is taken from the point (a, 0)
to the point (0, a) along a circular path of radius a about the origin in the x-y plane is :
[JEE-2013 ; 4/60]

2Kπ Kπ Kπ
(A) (B) (C) (D*) 0
a a 2a
Ans. (D)

Sol. suppose x = r cosθ

y = r sinθ

Rankers Varanasi office | Pandeypur Chauraha – Call 8601467389


K
force on particle is
r 3(r cos θ ˆi + r sin θˆj )
force is in radial direction so work done by this force along given path (circle) is zero.

5. A particle of mass 0.2 kg is moving in one dimension under a force that delivers a constant
power 0.5 W to the particle. If the initial speed (in ms–1) of the particle is zero, the speed (in
ms–1) after 5s is :

[JEE-2013 ; 4/60]

Ans. 5

1
Sol. E = P.t = 0.5W × 5s = 2.5 J = mv2 ⇒ v = 5 m/s
2
6.# Consider an elliptically shaped rail PQ in the vertical plane with OP = 3m and OQ= 4m. A block
of mass 1kg is pulled along the rail from P to Q with a force of 18 N, Which is always parallel
to line PQ (see the figure given). Assuming no frictional losses, the kinetic energy of the block
when it reaches Q is (n × 10) joules. The value of n is (take acceleration due to gravity = 10 ms–
2
)

[JEE (Advanced)-2014 ; P-1, 3/60]

Ans. 5

Sol. WF + Wg = Kf - K i

18×5 + 1g (-4) = Kf

90 - 40 = Kf

Kf = 50J = 5×10J

7*. A particle of mass m is initially at rest at the origin. It is subjected to a force and starts moving
along the x-axis. Its kinetic energy K changes with time as dK/dt = γt where λ is a positive
constant of appropriate dimensions. Which of the following statements is (are) true? JEE
(Advanced) 2018 ; P-2, 4/60, –2]

(A) The force applied on the particle is constant

(B) The speed of the particle is proportional to time

(C) The distance of the particle from the origin increases linearly with time

(D) The force is conservative

Rankers Varanasi office | Pandeypur Chauraha – Call 8601467389


Ans. (ABD)

dv dk
Sol. mv = = γt
dt dt

γ
v dv = t dt
m

v2 γ t2
= ⇒v ∝ t
2 m 2

dv
= constant ⇒ F = constant
dt

dx
∝ t ⇒ x ∝ t2
dt

Centre of Mass
CENTER OF MASS OF A SYSTEM OF 'N' DISCRETE PARTICLES
Consider a system of N point masses m1, m2, m3, ................ mn whose position vectors from
  
origin O are given by r1 , r2 , r3 ,............... respectively. Then the position vector of the center
of mass C of the system is given by
n


  
m1r1 + m2 r2 + ........ + mn rn 
∑m r
i=1
ii

rcm = ; rcm = n
m1 + m2 + ........ + mn
∑m
i=1
i

n
 1 
rcm =
M ∑m r
i=1
ii


where, mi ri is called the moment of mass of the particle w.r.t O.

Rankers Varanasi office | Pandeypur Chauraha – Call 8601467389


 n 
M=  ∑ mi  is the total mass of the system.
i = 1 
 

n

Note: If the origin is taken at the center of mass then ∑ m r = 0. Hence, the COM is the point
i=1
ii

about which the sum of “mass moments” of the system is zero.

CENTER OF MASS OF A CONTINUOUS MASS DISTRIBUTION


For continuous mass distribution the center of mass can be located by replacing summation
sign with an integral sign. Proper limits for the integral are chosen according to the situation

xcm =
∫ xdm , y cm =
∫ ydm , z cm =
∫ zdm
∫ dm ∫ dm ∫ dm
∫ dm = M (mass of the body)

 1 
rcm =
M ∫ rdm
Note: If an object has symmetric mass distribution about x axis then y coordinate of COM is
zero and
vice-versa

CENTER OF MASS OF A UNIFORM ROD


Suppose a rod of mass M and length L is lying along the x-axis with its one end at x = 0 and the
M
other at x = L. Mass per unit length of the rod =
L

M
Hence, dm, (the mass of the element dx situated at x = x is) = dx
L

The coordinates of the element dx are (x, 0, 0). Therefore, x-coordinate of COM of the rod will
be
L L M 
∫ xdm ∫ 0
(x)  dx 
L = 1 L L

0
xCOM = = xdx = dx

∫ dm M L 0 2
x=0 x=x x=L

The y-coordinate of COM is yCOM =


∫ ydm = 0. Similarly, z COM =0
∫ dm
L 
i.e., the coordinates of COM of the rod are  ,0,0  , i.e. it lies at the center of the rod.
2 

CENTER OF MASS OF A SEMICIRCULAR RING

Rankers Varanasi office | Pandeypur Chauraha – Call 8601467389


Figure shows the object (semi circular ring). By observation we can say that the x-coordinate
of the center of mass of the ring is zero as the half ring is symmetrical about y-axis on both
sides of the origin. Only we are required to find the y-coordinate of the center of mass.

Rdθ

y=Rsinθ
ycm dθ
θ X

1
To find ycm we use ycm =
M ∫ dmy .....(i)

Here for dm we consider an elemental arc of the ring at an angle θ from the x-direction of
angular width dθ. If radius of the ring is R then its y coordinate will be R sinθ, here dm is given
as

M
dm = × R dθ
πR

So from equation ....(i), we have


π π
1 M R
ycm =
M ∫ 0
πR
Rdθ (R sinθ) =
π ∫ sin θdθ
0

2R
ycm = .....(ii)
π

CENTER OF MASS OF SEMICIRCULAR DISC


Figure shows the half disc of mass M and radius R. Here, we are only required to find the y-
coordinate of the center of mass of this disc as center of mass will be located on its half vertical
diameter. Here to find ycm, we consider a small elemental ring of mass dm of radius x on the
disc (disc can be considered to be made up such thin rings of increasing radii) which will be
2M
integrated from 0 to R. Here dm is given as dm = (π x)dx
πR2

Y
ycm

dx

x
X
R

2x
Now the y-coordinate of the element is taken as , as in previous section, we have derived
π
2R
that the center of mass of a semi circular ring is concentrated at
π

Rankers Varanasi office | Pandeypur Chauraha – Call 8601467389


R R
1 2x 1 4M
Here ycm is given as ycm =
M ∫
0
dm
π
=
M ∫ πR
0
2
x 2 dx

4R
ycm =

CENTER OF MASS OF A SOLID HEMISPHERE


The hemisphere is of mass M and radius R. To find its center of mass (only y-coordinate), we
consider an element disc of width dy, mass dm at a distance y from the center of the
hemisphere. The radius of this elemental disc will be given as r = R2 − y 2

3M 3M
The mass dm of this disc can be given as dm = × πr2dy = (R2 – y2)dy
2πR3 2R3

R R R
1 1 3M 3
∫ ∫ 2R ∫ (R
2
ycm of the hemisphere is given as ycm = dmy = 3
(R2 – y2) dy y = − y 2 )ydy
M
0
M
0
2R3 0

3R
ycm =
8

CENTER OF MASS OF A HOLLOW HEMISPHERE


A hollow hemisphere of mass M and radius R. Now we consider an elemental circular strip of
angular width dθ at an angular distance θ from the base of the hemisphere. This strip will have
an area.

dS = 2πR cos θ Rdθ

Its mass dm is given as

M
dm = 2πR cos θ Rdθ
2πR2

Rankers Varanasi office | Pandeypur Chauraha – Call 8601467389


Here y-coordinate of this strip of mass dm can be taken as R sinθ. Now we can obtain the
center of mass of the system as.
π π
2 2
1 1  M 
ycm =
M ∫ dmR sin θ = M ∫  2πR
0 0
2
2πR2 cos θdθ  R sinθ

π
2
R

= R sin θ cos θdθ
0
⇒ ycm =
2

CENTER OF MASS OF A SOLID CONE


A solid cone has mass M, height H and base radius R. Obviously the center of mass of this cone
will lie somewhere on its axis, at a height less than H/2. To locate the center of mass we
consider an elemental disc of width dy and radius r, at a distance y from the apex of the cone.
Let the mass of this disc be dm, which can be given as

3M
dm = × πr2 dy
πR2H

H H H
1 1  3M  Ry 2  3 3H
∫ ∫ ∫ y dy =
3
here ycm can be given as ycm = ydm =  π dy y = 3
M M  πR2H  H   H 4
0 0  0

   
dr1 dr2 dr3 drn
m1 + m2 + m3 .............. + mn    

v cm = dt dt dt dt = m1v1 + m2 v 2 + m3 v 3 .......... + mn v n
M M

Here numerator of the right hand side term is the total momentum of the system i.e.,
summation of momentum of the individual component (particle) of the system

Hence velocity of center of mass of the system is the ratio of momentum of the system to the
mass of the system.
 
∴ PSystem = M v cm

Rankers Varanasi office | Pandeypur Chauraha – Call 8601467389


Acceleration of center of mass of system
   
dv1 dv 2 dv 3 dv n    
m1 + m2 + m3 .............. + mn
 dt dt dt dt m1a1 + m2a2 + m3 a3 .......... + mnan
acm = =
M M

Net forceonsystem Net External Force + Net internal Force Net External Force
= = =
M M M

( action and reaction both of an internal force must be within the system. Vector summation
will cancel all internal forces and hence net internal force on system is zero)
 
∴ Fext = M acm

where Fext is the sum of the 'external' forces acting on the system. The internal forces which
the particles exert on one another play absolutely no role in the motion of the center of mass.

If no external force is acting on a system of particles, the acceleration of center of mass of the
system will be zero. If ac = 0, it implies that vc must be a constant and if vcm is a constant, it
implies that the total momentum of the system must remain constant. It leads to the principal
of conservation of momentum in absence of external forces.
 
If Fext = 0 then = v cm constant

“If resultant external force is zero on the system, then the net momentum of the system
must remain constant”.

Motion of COM in a moving system of particles :

(1) COM at rest :

If Fext = 0 and Vcm = 0, then COM remains at rest. Individual components of the system may
move and have non-zero momentum due to mutual forces (internal), but the net
momentum of the system remains zero.

(i) All the particles of the system are at rest.

(ii) Particles are moving such that their net momentum is zero.

Example:

(iii) A bomb at rest suddenly explodes into various smaller fragments, all moving in
different directions then, since the explosive forces are internal & there is no external
force on the system for explosion therefore, the COM of the bomb will remain at the
original position and the fragment fly such that their net momentum remains zero.

Rankers Varanasi office | Pandeypur Chauraha – Call 8601467389


(iv) Two men standing on a frictionless platform, push each other, then also their net
momentum remains zero because the push forces are internal for the two men
system.

(v) A boat floating in a lake, also has net momentum zero if the people on it changes their
position, because the friction force required to move the people is internal of the boat
system.

(vi) Objects initially at rest, if moving under mutual forces (electrostatic or gravitation)also
have net momentum zero.

(vii) A light spring of spring constant k kept compressed between two blocks of masses m1
and m2 on a smooth horizontal surface. When released, the blocks acquire velocities in
opposite directions, such that the net momentum is zero.

(viii) In a fan, all particles are moving but COM is at rest

(2) COM moving with uniform velocity :

If Fext = 0, then Vcm remains constant therefore, net momentum of the system also remains
conserved. Individual components of the system may have variable velocity and
momentum due to mutual forces (internal), but the net momentum of the system remains
constant and COM continues to move with the initial velocity.

(i) All the particles of the system are moving with same velocity.

e.g.: A car moving with uniform speed on a straight road, has its COM moving with a
constant velocity.

(ii) Internal explosions / breaking does not change the motion of COM and net
momentum remains conserved. A bomb moving in a straight line suddenly
explodes into various smaller fragments, all moving in different directions then, since
the explosive forces are internal & there is no external force on the system for
explosion therefore, the COM of the bomb will continue the original motion and
the fragment fly such that their net momentum remains conserved.

Rankers Varanasi office | Pandeypur Chauraha – Call 8601467389


(iii) Man jumping from cart or buggy also exert internal forces therefore net momentum
of the system and hence, Motion of COM remains conserved.

(iv) Two moving blocks connected by a light spring on a smooth horizontal surface. If the
acting forces is only due to spring then COM will remain in its motion and momentum
will remain conserved.

(v) Particles colliding in absence of external impulsive forces also have their momentum
conserved.

(3) COM moving with acceleration :

If an external force is present then COM continues its original motion as if the external
force is acting on it, irrespective of internal forces.

Example:

Projectile motion : An axe thrown in air at an angle θ with the horizontal will perform a
complicated motion of rotation as well as parabolic motion under the effect of gravitation

u2 sin2 θ
Hcom =
2g

u2 sin 2θ 2u sin θ
Rcom = T=
g g

Example :

Circular Motion : A rod hinged at an end, rotates, than its COM performs circular motion.
The centripetal force (Fc) required in the circular motion is assumed to be acting on the
COM.

Fc = mω2RCOM

Rankers Varanasi office | Pandeypur Chauraha – Call 8601467389


Rcom

Fc
ω Fc
mg
Fc Fc

mg
mg
mg

IMPULSE

Impulse of a force F acting on a body for the time interval t = t1 to t = t2 is defined as :
    
t2 dv 
Ι = ∫ t1
Fdt ⇒ Ι = ∫ Fdt = ∫ m
dt
dt = ∫ mdv
   
Ι = m( v 2 - v1 ) = ΔP = change in momentum due to force

 t2  
Also, ΙRe s = ∫ t1
FRe s dt = ΔP (impulse - momentum theorem)

Note : Impulse applied to an object in a given time interval can also be calculated from the
area under force time (F-t) graph in the same time interval.

Instantaneous Impulse :

There are many cases when a force acts for such a short time that the effect is instantaneous,
e.g., a bat striking a ball. In such cases, although the magnitude of the force and the time for
which it acts may each be unknown but the value of their product (i.e., impulse) can be known
by measuring the initial and final momenta. Thus, we can write.
    

Ι = F dt = ∆P = Pf − Pi

Important Points :
(1) It is a vector quantity.

(2) Dimensions = [MLT–1]

(3) SΙ unit = kg m/s

(4) Direction is along change in momentum.

Rankers Varanasi office | Pandeypur Chauraha – Call 8601467389


(5) Magnitude is equal to area under the F-t. graph.
   

(6) Ι = Fdt = Fav dt = Fav ∆t ∫
(7) It is not a property of a particle, but it is a measure of the degree to which an external
force changes the momentum of the particle.

Impulsive force :
A force, of relatively higher magnitude and acting for relatively shorter time, is called
impulsive force.

An impulsive force can change the momentum of a body in a finite magnitude in a very short
time interval. Impulsive force is a relative term. There is no clear boundary between an
impulsive and
Non-Impulsive force.

Note : Usually colliding forces are impulsive in nature.

Since, the application time is very small, hence, very little motion of the particle takes place.

Important points :
1. Gravitational force and spring force are always non-Impulsive.

2. Normal, tension and friction are case dependent.

3. An impulsive force can only be balanced by another impulsive force.

1. Impulsive Normal : In case of collision, normal forces at the surface of collision are always
impulsive

eg. Ni = Impulsive; Ng = Non-impulsive

N1

N2 Both normals are Impulsive


/////////////////////////////

N1
N1 N1, N3 = Impulsive; N2 = non-impulsive
//////////////////////////////////////////////////////////////////
N3
N2

Rankers Varanasi office | Pandeypur Chauraha – Call 8601467389


Both normals are Impulsive

2. Impulsive Friction : If the normal between the two objects is impulsive, then the friction
between the two will also be impulsive.

Friction at both surfaces is impulsive

Friction due to N2 is non-impulsive and due


to N3

and N1 are impulsive.

3. Impulsive Tensions : When a string jerks, equal and opposite tension act suddenly at each
end. Consequently equal and opposite impulses act on the bodies attached with the string
in the direction of the string. There are two cases to be considered.

(a) One end of the string is fixed : The impulse which acts at the fixed end of the string
cannot change the momentum of the fixed object there. The object attached to the
free end however will undergo a change in momentum in the direction of the string.
The momentum remains unchanged in a direction perpendicular to the string where
no impulsive forces act.

(b) Both ends of the string attached to movable objects : In this case equal and opposite
impulses act on the two objects, producing equal and opposite changes in
momentum. The total momentum of the system therefore remains constant,
although the momentum of each individual object is changed in the direction of the
string. Perpendicular to the string however, no impulse acts and the momentum of
each particle in this direction is unchanged.

Rankers Varanasi office | Pandeypur Chauraha – Call 8601467389


///////////////////

T is Impulsive T
A

T is non-impulsive B

C
T is non-impulsive

All normal are impulsive but tension


T is impulsive only for the ball A

For this example : In case of rod, Tension is always impulsive and in case of spring,
Tension is always non-impulsive.

COLLISION OR IMPACT

Collision is an event in which an impulsive force acts between two or more bodies for a short
time, which results in change of their velocities.

Note :

(a) In a collision, particles may or may not come in physical contact.

(b) The duration of collision, ∆t is negligible as compared to the usual time intervals of
observation of motion.

(c) In a collision the effect of external non impulsive forces such as gravity are not taken into
a account as due to small duration of collision (∆t) average impulsive force responsible for
collision is much larger than external forces acting on the system.

The collision is infact a redistribution of total momentum of the particles. Thus, law of
conservation of linear momentum is indispensable in dealing with the phenomenon of collision
between particles.

Line of Impact

The line passing through the common normal to the surfaces in contact during impact is called
line of impact. The force during collision acts along this line on both the bodies.

Direction of Line of impact can be determined by:

(a) Geometry of colliding objects like spheres, discs, wedge etc.

(b) Direction of change of momentum.

If one particle is stationary before the collision then the line of impact will be along its motion
after collision.

Classification of collisions

(a) On the basis of line of impact

Rankers Varanasi office | Pandeypur Chauraha – Call 8601467389


(i) Head-on collision : If the velocities of the colliding particles are along the same line
before and after the collision.

(ii) Oblique collision : If the velocities of the colliding particles are along different lines
before and after the collision.

(b) On the basis of energy :

(i) Elastic collision : In an elastic collision, the colliding particles regain their shape and
size completely after collision. i.e., no fraction of mechanical energy remains stored
as deformation potential energy in the bodies. Thus, kinetic energy of system after
collision is equal to kinetic energy of system before collision. Thus in addition to the
linear momentum, kinetic energy also remains conserved before and after collision.

(ii) Inelastic collision : In an inelastic collision, the colliding particles do not regain their
shape and size completely after collision. Some fraction of mechanical energy is
retained by the colliding particles in the form of deformation potential energy. Thus,
the kinetic energy of the particles after collision is not equal to that of before collision.
However, in the absence of external forces, law of conservation of linear momentum
still holds good.

(iii) Perfectly inelastic : If velocity of separation along the line of impact just after collision
becomes zero then the collision is perfectly inelastic. Collision is said to be perfectly
inelastic if both the particles stick together after collision and move with same
velocity,

Note : Actually collision between all real objects are neither perfectly elastic nor perfectly
inelastic, its inelastic in nature.

Examples of line of impact and collisions based on line of impact


(i) Two balls A and B are approaching each other such that their centers are moving along
line CD.

Head on Collision

(ii) Two balls A and B are approaching each other such that their center are moving along
dotted lines as shown in figure.

Rankers Varanasi office | Pandeypur Chauraha – Call 8601467389


Oblique Collision

(iii) Ball is falling on a stationary wedge.

COEFFICIENT OF RESTITUTION (e)

The coefficient of restitution is defined as the ratio of the impulses of reformation and
deformation of either body.

e=
Impulse of reformation
=
∫ F dt
r

Impulse of deformation
∫ F dt
d

Velocity of seperation along line of impact


=
Velocity of approach along line of impact

The most general expression for coefficient of restitution is

velocity of separation of points of contact along line of impact


e=
velocity of approach of point of contact along line of impact

Example for calculation of e

Two smooth balls A and B approaching each other such that their centers are moving along
line CD in absence of external impulsive force. The velocities of A and B just before collision
be u1 and u2 respectively. The velocities of A and B just after collision be v1 and v2 respectively.

Just Before collision Just After collision


u1 u2 v1 v2

Line of impact Line of impact


C D C D
A B A B

Rankers Varanasi office | Pandeypur Chauraha – Call 8601467389


u12> u v12< v
u2 v v2
u1 v v1
ND ND NR
NR
m1 m1 m1
m2 m2 m2

  Deformation Reformation

 Fext = 0 momentum is conserved for the system.


⇒ m1u1 + m2 u2 = (m1 + m2)v = m1v1 + m2v2

m1u1 + m2u2 m v + m2 v 2
⇒ v= = 1 1 .......(1)
m1 + m2 m1 + m2

Impulse of Deformation :

JD = change in momentum of any one body during deformation.

= m2 (v – u2) for m2

= m1 (–v + u1) for m1

Impulse of Reformation :

JR = change in momentum of any one body during Reformation.

= m2 (v2 – v) for m2

= m1 (v – v1) for m1

Impulse of Reformation(JR ) v −v Velocity of separation along line of impact
e=  = 2 1 =
Impulse of Deformation(JD ) u1 − u2 Velocity of approach along line of impact

Note : e is independent of shape and mass of object but depends on the material. The
coefficient of restitution is constant for a pair of materials.

(a) e = 1 Impulse of Reformation = Impulse of Deformation

Velocity of separation = Velocity of approach

Kinetic energy of particles after collision may be equal to that of before collision.

Collision is elastic.

(b) e = 0 Impulse of Reformation = 0

Velocity of separation = 0

Kinetic energy of particles after collision is not equal to that of before collision.

Rankers Varanasi office | Pandeypur Chauraha – Call 8601467389


Collision is perfectly inelastic.

(c) 0 < e < 1 Impulse of Reformation < Impulse of Deformation

Velocity of separation < Velocity of approach

Kinetic energy of particles after collision is not equal to that of before collision.

Collision is Inelastic.

Note : In case of contact collisions e is always less than unity.

∴ 0≤e≤1

Important Point :

In case of elastic collision, if rough surface is present then kf < ki (because friction is
impulsive).

Where, k is Kinetic Energy.

Rough
/////////////////////////////////////

A particle ‘B’ moving along the dotted line collides with a rod also in state of motion as shown
in the figure. The particle B comes in contact with point C on the rod.

To write down the expression for coefficient of restitution e, we first draw the line of impact.
Then we resolve the components of velocities of points of contact of both the bodies along
line of impact just before and just after collision.

v 2x − v1x
Then e=
u1x − u2x

Collision in one dimension (Head on)


m1 m2 m1 m2
u1 u2 v1 v2

(a) (b)
Before Collision After Collision

Rankers Varanasi office | Pandeypur Chauraha – Call 8601467389


v 2 − v1
e= ⇒ (u1 – u2)e = (v2 – v1)
u1 − u2

By momentum conservation, m1u1 + m2u2 = m1v1 + m2v2


m u + m2u2 − m2 e(u1 − u2 )
v2 = v1 + e(u1 – u2) and v1 = 1 1
m1 + m2

m1u1 + m2u2 + m1e(u1 − u2 )


v2 =
m1 + m2

Special Case :

(1) e = 0

⇒ v1 = v 2

⇒ for perfectly inelastic collision, both the bodies, move with same vel. after collision.

(2) e = 1

and m1 = m2 = m,

we get v1 = u2 and v2 = u1

i.e., when two particles of equal mass collide elastically and the collision is head on, they
exchange their velocities., e.g.

v=0
1

2m/s m
m

Before Collision

(3) m1 >> m2

m2
m1 + m2 = m1 and =0
m1

⇒ v1 = u1 No change and v2 = u1 + e(u1 – u2)

Collision in two dimension (oblique)


1. A pair of equal and opposite impulses act along common normal direction. Hence, linear
momentum of individual particles do change along common normal direction. If mass of
the colliding particles remain constant during collision, then we can say that linear velocity
of the individual particles change during collision in this direction.

2. No component of impulse act along common tangent direction. Hence, linear momentum
or linear velocity of individual particles (if mass is constant) remain unchanged along this
direction.

Rankers Varanasi office | Pandeypur Chauraha – Call 8601467389


3. Net impulse on both the particles is zero during collision. Hence, net momentum of both
the particles remain conserved before and after collision in any direction.

4. Definition of coefficient of restitution can be applied along common normal direction, i.e.,
along common normal direction we can apply

Relative speed of separation = e (relative speed of approach)

Q-01 . Two spheres are moving towards each other. Both have same radius but their masses are 2
kg and 4 kg. If the velocities are 4 m/s and 2 m/s respectively and coefficient of
restitution is
e = 1/3, find.

(a) The common velocity along the line of impact. 2kg


4m/s
A
(b) Final velocities along line of impact. R
2m/s R

(c) Impulse of deformation. B


4kg

(d) impulse of reformation.

(e) Maximum potential energy of deformation.

(f) Loss in kinetic energy due to collision.

Solution :

A 4m/s C Line of motion


2kg θ
R R
R 4kg
Line of motion 2m/s B
Line of impact

BC R 1
In ∆ABC sin θ = = = or θ = 30º
AB 2R 2

(a) By conservation of momentum along line of impact.

4sin30º

2kg 4m/s
30º

4cos30º 4sin30º
2cos30º
30º
2m/s B 4kg
v
2sin30º 2sin30º

Just Before Collision Along LOI Maximum Deformed State

2(4 cos 30º) – 4(2cos 30º) = (2 + 4)v

Rankers Varanasi office | Pandeypur Chauraha – Call 8601467389


or v = 0 (common velocity along LOΙ)

(b)

4sin30º

A
2kg v1
4kg B
v2
2sin30º

Just After Collision Along LOI

Let v1 and v2 be the final velocity of A and B respectively then, by conservation of


momentum along line of impact,

2(4cos 30º) – 4(2cos 30º) = 2(v1) + 4(v2)

or 0 = v1 + 2v2 .........(1)

By coefficient of restitution,

velocity of separation along LOΙ


e=
velocity of approach along LOΙ

1 v 2 − v1
or = or v2 – v1 = 3 ........ (2)
3 4 cos30º +2cos30º

from the above two equations,

−2 1
v1 = m / s and v2 = m/s
3 3

(c) JD = m1(v – u1) = 2(0 – 4 cos 30º) = – 4 3 N-s

1 4
(d) JR = eJD = (–4 3 ) = – N-s
3 3

(e) Maximum potential energy of deformation is equal to loss in kinetic energy during
deformation upto maximum deformed state,

1 1 1
U= m1(u1 cos θ)2 + m2(u2 cos θ)2 – (m1 + m2)v2
2 2 2

1 1 1
= 2(4 cos 30º)2 + 4(–2cos 30º)2 – (2 + 4) (0)2 or U = 18 Joule
2 2 2

1 1
(f) Loss in kinetic energy, ∆KE = m1(u1 cos θ)2 + m2(u2 cos θ)2 –
2 2
1 2 1 2
 2 m1v1 + 2 m2 v 2 
 

Rankers Varanasi office | Pandeypur Chauraha – Call 8601467389


1 1  1  2 2 1  1 2 
= 2(4 cos 30º)2 + 4(–2 cos 30º)2 –  2   + 4  
2 2 2  3  2  3 
 

∆KE = 16 Joule

VARIABLE MASS SYSTEM : ( ADVANCED SPECIAL )



If a mass is added or ejected from a system, at rate µ kg/s and relative velocity v rel (w.r.t. the

system), then the force exerted by this mass on the system has magnitude µ v rel .

Thrust Force ( Ft )

   dm 
Ft = v rel  
 dt 

Suppose at some moment t = t mass of a body is m and its velocity is v . After some time at
 
t = t + dt its mass becomes (m – dm) and velocity becomes v + dv . The mass dm is ejected
  
with relative velocity v r . Absolute velocity of mass ‘dm’ is therefore ( v + v r ). If no external
forces are acting on the system, the linear momentum of the system will remain conserved,
 
or Pi = Pf
    
or m v = (m – dm) ( v + d v ) + dm ( v + v r )
      
or m v = m v + md v – (dm) v – (dm) (d v ) + (dm) v + v r dm

The term (dm) (d v ) is too small and can be neglected.

   dv    dm 
∴ md v = – v r dm or m   = v r  − 
 dt   dt 

 dv  
Here, m −  = thrust force Ft
 dt 
( )
dm
and – = rate at which mass is ejecting
dt

   dm 
or Ft = v r  
 dt 

Problems related to variable mass can be solved in following four steps


1. Make a list of all the forces acting on the main mass and apply them on it.

   dm 
2. Apply an additional thrust force Ft on the mass, the magnitude of which is vr  ± 
 dt 

and direction is given by the direction of v r in case the mass is increasing and otherwise

the direction of – v r if it is decreasing.

Rankers Varanasi office | Pandeypur Chauraha – Call 8601467389


 
dv
3. Find net force on the mass and apply Fnet = m (m = mass at the particular instant)
dt

4. Integrate it with proper limits to find velocity at any time t.

Note : Problems of one-dimensional motion (which are mostly asked in JEE) can be solved in
easier manner just by assigning positive and negative signs to all vector quantities. Here are
few example in support of the above theory.

Example 2. A flat car of mass m0 starts moving to the right due to a constant horizontal force F. Sand
spills on the flat car from a stationary hopper. The rate of loading is constant and equal
to µ kg/s. Find the time dependence of the velocity and the acceleration of the flat
car in the process of loading. The friction is negligibly small.

µ
m0 F

Solution : Initial velocity of the flat car is zero. Let v be its velocity at time t and m its mass at
that instant. Then

At t = 0, v = 0 and m = m0 at t = t, v = v and m = m0 + µt

Here, vr = v (backwards)

dm

dt

dm
∴ Ft = vr = µv (backwards)
dt

Net force on the flat car at time t is Fnet = F – Ft

dv
or m = F – µv ....(i)
dt

dv
or (m0 + µt) = F – µv
dt

Rankers Varanasi office | Pandeypur Chauraha – Call 8601467389


v dv t dt
or ∫ 0 F − µv
= ∫ 0 m0 + µt

1 1
∴ – [n (F – µv)]0v = [n (m0 + µt)]0t
µ µ

 F   m0 + µt 
⇒ n   = n  
 F − µv   m0 

F m + µt Ft
∴ = 0 or v= Ans.
F − µv m0 m0 + µt

dv
From Eq. (i), = acceleration of flat car at time t
dt

F − µv
or =
m

 Fµt 
 F − m + µt  Fm0
a=  0  or a= Ans.
 m0 + µt  (m0 + µt)2
 
 

Rocket propulsion : ( ADVANCED SPECIAL )

Let m0 be the mass of the rocket at time t = 0. m its mass at any time t and v its velocity at that
moment. Initially, let us suppose that the velocity of the rocket is u.

 −dm 
Further, let   be the mass of the gas ejected per unit time and vr the exhaust velocity
 dt 
 −dm 
of the gases with respect to rocket. Usually   and vr are kept constant throughout the
 dt 
journey of the rocket. Now, let us write few equations which can be used in the problems of
rocket propulsion. At time t = t,

 −dm 
1. Thrust force on the rocket Ft = vr   (upwards)
 dt 

2. Weight of the rocket W = mg (downwards)

3. Net force on the rocket Fnet = Ft – W (upwards)

Rankers Varanasi office | Pandeypur Chauraha – Call 8601467389


 −dm 
or Fnet = vr   – mg
 dt 

F
4. Net acceleration of the rocket a=
m

dv v  −dm 
or = r  –g
dt m  dt 

vr
or dv = (–dm) – g dt
m

v m −dm t
or ∫ u
dv = v r ∫ m0 m
–g ∫ 0
dt

 m0 
Thus, v = u – gt + vr n   ...(i)
 m 

Note :

 dm  dm
1. Ft = vr  −  is upwards, as vr is downwards and dt is negative.
 dt 

2. If gravity is ignored and initial velocity of the rocket u = 0, Eq. (i) reduces to v = vr ln
 m0 
 
 m 

Rankers Varanasi office | Pandeypur Chauraha – Call 8601467389


Advanced Level Problems
1. A uniform rope of linear mass density λ and length  is coiled on a smooth horizontal surface.
One end is pulled up with constant velocity v. Then find average power applied by the external
agent in pulling the entire rope just off the ground?

2. In the above question the maximum power delivered by the agent in pulling up the rope is

3. A particle is projected from point O on the ground with velocity


u = 5 5 m/s at angle α = tan–1 (0.5). It strikes at a point C on a
fixed smooth plane AB having inclination of 37º with horizontal
as shown in figure. If the particle does not rebound, calculate.

(a) coordinates of point C in reference to coordinate system as


shown in the figure.

(b) maximum height from the ground to which the particle


rises. (g = 10 m/s2)

4. Two blocks A & B of mass ‘m’ & 2m respectively are joined to


the ends of an undeformed massless spring of spring
constant ' k '. They can move on a horizontal smooth surface.
Initially A & B have velocities ' u ' towards left and '2 u'
towards right respectively.

Constant forces of magnitudes F and 2 F are always acting on A and B respectively in the
directions shown. Find the maximum extension in the spring during the motion.

5. Two identical buggies 1 and 2 with one man in each move without friction due to inertia along
the parallel rails toward each other. When the buggies got opposite to each other, the men
exchange their places by jumping in the direction perpendicular to the direction of motion. As
aconsequence,buggy 1 stops and buggy 2 keeps moving in the same direction, with its velocity

Rankers Varanasi office | Pandeypur Chauraha – Call 8601467389


becoming equal to v. Find the initial velocities of the buggies v1 and v2 if the mass of each
buggy (without a man) equals M and the mass of each man is m.

6. Two identical buggies move one after the other due to inertia (without friction) with the same
velocity v0. A man of mass m rides the rear buggy. At a certain moment the man jumps into
the front buggy with a velocity u relative to his buggy. Knowing that the mass of each buggy
is equal to M, find the velocities with which the buggies will move after that.

7. Two men, each of mass m, stand on the edge of a stationary buggy of mass M. Assuming the
friction to be negligible, find the velocity of the buggy after both men jump off with the same
horizontal velocity u relative to the buggy: (i) simultaneously; (ii) one after the other. In what
case will the velocity of the buggy be greater and how many times?

8. A stationary pulley carries a rope whose one end supports a ladder with a man and the other
end the counterweight of mass M. The man of mass m climbs up a distance ' with respect to
the ladder and then stops. Neglecting the mass of the rope and the friction in the pulley axle,
find the displacement
 of the centre of inertia of this system.

9. A particle of mass m1 experienced a perfectly elastic collision with a stationary particle of mass
m2. What fraction of the kinetic energy does the striking particle lose, if

(a) it recoils at right angles to its original motion direction;

(b) the collision is a head-on one?

10. Body 1 experiences a perfectly elastic collision with a stationary Body 2. Determine their mass
ratio, if

(a) after a head-on collision the particles fly apart in the opposite directions with equal,
velocities;

(b) the particles fly apart symmetrically relative to the initial motion direction of particle 1
with the angle of divergence θ = 60°.

11. A ball moving transitionally collides with another stationary ball of the same mass. At the
moment of impact the angle between the straight line passing through the centres of the balls
and the direction of the initial motion of the striking ball is equal to α = 45°. Assuming the balls
to be smooth, find the fraction η of the kinetic energy of the striking ball that turned into
potential energy at the moment of the maximum deformation.

12. Particle 1 moving with velocity v = 10 m/s experienced a head-on collision with a stationary
particle 2 of the same mass. As a result of the collision, the kinetic energy of the system
decreased by η = 1.0%. Find the magnitude and direction of the velocity of particle 1 after the
collision.

Rankers Varanasi office | Pandeypur Chauraha – Call 8601467389


13. A particle of mass m having collided with a stationary particle of mass M deviated by an angle
π/2 whereas the particle M start moving at an angle θ = 30° to the direction of the initial
motion of the particle m. How much (in percent) and in what way has the kinetic energy of
this system changed after the collision, if M / m = 5.0 ?

14. A closed system consists of two particles of masses m1 and m2 which move at right angles to
each other with velocities v1 and v2 Find:

(a) the momentum of each particle and

(b) the total kinetic energy of the two particles in the reference frame fixed to their centre of
inertia.

15. A particle of mass m1 collides elastically with a stationary particle of mass m2 (m1 > m2). Find
the maximum angle through which the striking particle may deviate as a result of the collision.

16. Three identical discs A, B, and C as shown in figure rest on a smooth horizontal plane.
The disc A is set in motion with velocity v after which it experiences an elastic
collision simultaneously with discs B and C. The distance between the centres of the
latter discs prior to the collision is η times the diameter of each disc Find the velocity
of the disc A after the collision. At what value of η will the disc A recoil after the
collision; stop; move on?

17. A spaceship of mass m0 moves in the absence of external ' forces with a constant velocity v0.
To change the motion direction a jet engine is switched on. It starts ejecting a gas jet with
velocity u, which is constant relative to the spaceship and directed at right angles to the
spaceship motion. The engine is shut down when the mass of the spaceship decreases to m.
Through what angle α did the motion direction of the spaceship deviate due to the jet engine
operation?

18. Two smooth spheres of the same radius, but which have different masses
m1 & m2 collide inelastically. Their velocities before collision are 13 m/s
& 5 m/s respectively along the directions shown in the figure in which
cotθ = 5/12. An observer S′ moving parallel to the positive y - axis with a
constant speed of 5m/s observes this collision. He finds the final velocity
of m1 to be 5m/s along the y-direction and the total loss in the kinetic
energy of the system to be 1/72 of its initial value. Determine;

(a) The ratio of the masses

(b) The velocity of m2 after collision with (respect to observer)

Rankers Varanasi office | Pandeypur Chauraha – Call 8601467389


19. A cart of total mass M0 is at rest on a rough horizontal road. It ejects
bullets at rate of λ kg/s at an angle θ with the horizontal and at velocity
‘u’ (constant) relative to the cart. The coefficient of friction between the
cart and the ground is µ. Find the velocity of the cart in terms of time
‘ t ’. The cart moves with sliding.

20. A bullet of mass M is fired with a velocity 50 m/s at an angle with the horizontal.
At the highest point of its trajectory, it collides head on with a bob of mass 3M
10
suspended by a massless rod of length m and gets embedded in the bob.
3
After the collision the rod moves through an angle 120°. Find

(a) The angle of projection. [JEE - 1998]

(b) The vertical and horizontal coordinates of the initial position of the bob
with respect to the point of firing of the bullet. (g = 10m/s2)

21. A steel ball falling vertically strikes a fixed rigid plate A with velocity v0
and rebounds horizontally. The ball then strikes a second fixed rigid
plate B and rebounds vertically as shown. Assuming smooth surface and
the effect of gravity on motion of ball is to be neglected. Determine

(a) The required angles α and β.

(b) The magnitude of the velocity v1 & v2. Consider coefficient of


restitution for both plates as e.

22. A small spherical ball undergoes an elastic collision with a


rough horizontal surface. Before the collision, it is moving
at an angle θ to the horizontal (see Fig). You may assume
that the frictional force obeys the law f = µN during the
contact period, where N is the normal reaction on the ball
and µ is the coefficient of friction.

(a) Obtain θm (µ) so that the subsequent horizontal range of the ball after leaving the horizontal
surface is maximized.

(b) Find the allowed range for θm.

23. Two skaters (A and B), each of mass 70 kg, are


approaching each another on a frictionless surface,
each with a speed of 1 ms–1. Skater A carries a ball of
mass 10 kg. Both skaters can toss the ball at 5 ms–1
relative to themselves such that when A tosses the
ball at t = 0 s to B then the ball leaves at 6 ms–1 with
respect to the ground.

Rankers Varanasi office | Pandeypur Chauraha – Call 8601467389


Further, they start (t = 0s) tossing the ball back and forth when they are 10m apart (see Fig.
(1)). Assume that the motion is one dimensional, all collisions are completely inelastic and that
the time delay between receiving the ball and tossing it back is 1s.

(a) State initial momenta of skaters (just before t = 0 s).


 
PA = ; PB =

(b) At t = 0 s skater A tosses the ball to skater B. State momenta of both the skaters
immediately after B catches the ball.
 
PA = ; PB =

(c) Indicates the minimum number of tosses by each skater required to avoid collision.

Number of tosses by A = ; Number of tosses by B =

(d) Indicate motion of each skater on the following x–t plot if no tosses are made. [Note : For this
and next part you must select the scale on the time axis approximately. You may use a pencil
for sketching].

(e) Indicate motion of each skater on the following x–t plot from t = 0 s till just after one round
trip by the ball (from A to B and back to A).

24. A neutron is scattered through (≡ deviation from its original direction) θ degree in an elastic
2
collision with an initially stationary deutron. If the neutron loses of its initial K.E. to the
3
deutron then find the value of θ. (In atomic mass unit, the mass of a neutron is 1u and mass
of a deutron is 2u).

Rankers Varanasi office | Pandeypur Chauraha – Call 8601467389


25. A shell flying with velocity v = 500 m/s bursts into three identical fragments so that the kinetic
energy of the system increases η = 1.5 times. What maximum velocity can one of the
fragments obtain?

26. A particle moves along a closed trajectory in a central field of force where the particle's
potential energy U = kr2 (k is positive constant, r is the distance of the particle from the centre
O of the field). Find the mass of the particle if its minimum distant; from the point O equals r1
and its velocity at the point farthest from O equals v2-

27. This problem is designed to illustrate the advantage that can be 1st Stage 2nd Stage Pay
obtained by the use of multiple-staged instead of single-staged load

rockets as launching vehicles. Suppose that the payload (e.g., a


space capsule) has mass m and is mounted on a two-stage m
rocket (see figure). The total mass (both rockets fully fuelled, Nm
nm

plus the payload) is Nm.

The mass of the second-stage rocket plus the payload, after first-stage burnout and
separation, is nm. In each stage the ratio of container mass to initial mass (container plus fuel)
is r, and the exhaust speed is V, constant relative to the engine. Note that at the end of each
state when the fuel is completely exhausted, the container drops off immediately without
affecting the velocity of rocket. Ignore gravity.

(a) Obtain the velocity v of the rocket gained from the first-stage burn, starting from rest in terms
of {V,N,n,r}

(b) Obtain a corresponding expression for the additional velocity u gained from the second
stage burn.

(c) Adding v and u, you have the payload velocity w in terms of N, n, and r. Taking N and r as
constants, find the value of n for which w is a maximum. For this maximum condition
obtain u/v.

(d) Find an expression for the payload velocity ws of a single-stage rocket with the same values of
N, r, and V

(e) Suppose that it is desired to obtain a payload velocity of 10 km/s, using rockets which V =
2.5km/s and r = 0.1. Using the maximum condition of part (c) obtain the value of N if the
job is to be done with a two-stage rocket. [Physics
Olympiad (STAGE-2) - 2016]

Answer Key

λvg
1. + λv 3 2. λgv + v 3 λ 3. (a) (5m, 1.25m) (b)
2
4.45 m.

4F + 16F2 + 54mu2 k
4. xmax = 5. v1 = – mv / (M – m), v2 = Mv/
3k
(M – m)

Rankers Varanasi office | Pandeypur Chauraha – Call 8601467389


m mMu
6. vrear = v0 – u; vfront = v0 +
M+m (M + m)2
 
– 2mu –m(2M + 3m)u (2M + 3m)
7. (i) v1 = (ii) v2 = , v2 > v1 by a factor of
(M + 2m) (M + m)(M + 2m) (2M + 2m)

4 m1 m2
8.  = 'm / 2M 9. (a) η = 2m1/(m1 + m2); (b) η =
( m1 + m2 )2

m1 m1 1
10. (a) = 1/3; (b) = 1 + 2cosθ = 2.0 11. η= cos2α = 0.25
m2 m2 2

12. Will continue moving in the same direction, although this time with the velocity v’ = (1 –
1 − 2η )v/2. For η << 1 the velocity v’ = ηv/2 = 5 cm/s.

13. ∆T/ T = (1 + m/M) tan2θ + m/M – 1 = –40%

14. (a) p = µ v12 + v 22 ; (b) T = 1/2µ( v12 + v 22 ) . Here µ = m1m2/(m1 + m2)

15. sinθmax = m2/m1

16. v’ = –v (2 – η2) / (6 – η2). Respectively at smaller η, equal, or greater than 2

m1 9
17. α = (u/u0) In (m0 / m) 18. (a) = (b) 9 m/s
m2 13

 M0 
19. v = (ucos θ − µusin θ)n   − µgt 20. (a) 37º, (b) x = 120 m and y =
 (M0 − λt) 
45 m

21. tanα = e , ν1 = e ν0, cotβ = e , ν2 = eν0

22. From fig. vy = vsinθ

Note that the surface is rough and there is frictional force along the x-direction. Hence elastic
collision does not constrain the velocity along the x-direction. it implies that the y-component
of the velocity vsinθ changes only in sign. From Newton’s second law along vertical y-direction,
change in momentum is given by the linear impulse, which yields :

2mvsinθ = Ndt ∫ ...(i)

From newton’s second law along x-direction, we have


mvcosθ – µ Ndt = mvx ...(ii)

Inserting Eq. (i) in Eq. (ii) we obtain

vx = v(cos(θ) – 2µsin(θ))

Rankers Varanasi office | Pandeypur Chauraha – Call 8601467389


Figure : Free body diagram of the ball in contact with the floor.

2v y
Range = vx × time of flight = vx ×
g

2v 2
R(θ) = f(θ)
g

where f(θ) = sinθ(cosθ – 2µsinθ)

To maximize R, set f ’(θm) = 0

1
which yields θm = cot −1(2µ )
2

(b) Possible range of θm : θm∈]0, π/4[

     
23. (a) PA = 80 i kg. m.s–1 or Pa = 70 i kg.m.s–1 ; Pb = – 70 i kg.m.s–1
     
(b) PA = 20 i kg.m.s–1 ; Pb = – 10 i kg.m.s–1 or Pb = – 70 / 8 i kg.m.s–1

(c) A = 1, B = 1

(d) See Fig. (1)

(e) See Fig. (2)

Rankers Varanasi office | Pandeypur Chauraha – Call 8601467389


24. 90 25. vmax = v(1 + 2(η − 1) ) = 1.0 km per second 26. m =
2kr12/v22.

dv  dm
27. (a) Variable mass equation gives=
m Fexternal + Vrelative
dt dt
 
No gravity hence Fexternal = 0 , v relative = v . Solving rocket equation

mi
= v = V ln .....(1)
mf

Here initially mass mi = Nm .....(2)

Final mass mf = [Nr + n(1 – r)]m .....(3)

(b) Now mi = nm, mf = m(nr + 1 – r). Equation (1) yields


n
u = V ln .....(4)
nr + 1– r
(c) From equation (3 to 4)

Nn
ω = V ln
[Nr + n(1– r)] [nr + 1– r]

V ln f(n)

Maximizing ω is equivalent to maximizing f(n). Differentiating and setting equal to zero, we


obtain

u ln[ N /{r N + (1 − r)}]


n= N⇒
= = 1 .....(5)
v ln[N / Nr + N(1 − r)}]

where we have used equation (1 and 4)

N
(d) Here mi = Nm and mf = m + r(Nm – m). Using equation ws = V ln
Nr + 1 − r

N
(e) Payload velocity w = u + v = 2V ln
r N + 1− r

For the desired value of w, N = 649.4. Answer should be an integer number for the number of
state. Hence N = 650.

Rankers Varanasi office | Pandeypur Chauraha – Call 8601467389


SOLUTION
1. Sol. Fnet =
d
dt
(mv)

dm
F–mg = v+0
dt

dm
F = mg + v , (m = vtλ)
dt

= vtλg + v2λ

P = Fv = v2 λgt + λv3
T

∫ Pdt
0 T
<P> = = v2λg   + λv3 [ T = /v]
T 2

λvg
<P> = + λv3
2

d
2. Sol. Fnet = (mv)
dt

dm
F–mg = v+0
dt

Rankers Varanasi office | Pandeypur Chauraha – Call 8601467389


dm
F = mg + v , (m = vtλ)
dt

= vtλg + v2λ


P = Fv = v2 λgt + λv3, P ↑ as t↑ so Pmax = v2λg   + λv3 = λgv + λv3
v

3. Sol. Let (x, y) be the coordinates of point C.

Now x component of point C = OA + AD

10 10 + 4y
∴ x= + y cot 37º = ....(1)
3 3

As point C lies on the trajectory of a parabola,

g x2
we have vr% y = x tan α –
2 u2 cos2 α

Given that, α = 0.5 and hence cos α = 2/ 5

Solving eqs. (1) and (2), we get x = 5 m and y = 1.25 m.

Hence the coordinates of point C are (5m, 1.25m)

(b) Let υy be the vertical component of velocity of the particle just before collsion at C. using
v = u + a t, we have

υy = u sin α – g (x/u cos α) ( t = x/u cos α)

= 5 5 × (1 × 5 ) – 10 × 5/5 5 cos α

5 5 10 × 5
= – =0
5 (5 5 × 2 / 5 )

Thus at C, the particle has only horizontal component of velocity

ux = u cos a = 5 5 × (2 × 5 ) = 10 m/s

Given that the particle does not rebound after collision. So, the normal component of velocity
(normal to the plane AB) becomes zero. Now, the particle slides up the plane due to tangential
component ux cos 37º = 8 m/s.

Rankers Varanasi office | Pandeypur Chauraha – Call 8601467389


Let h be the height raised of the particle. Then

1
mgh= m (8)2 or h = 3.2 m
2

Height of the particle from the ground = y + h

∴ H = 1.25 + 3.2 = 4.45 m.

Sol.

4. Solution

2F–F F – mu + (2m). (2u)


acm = = Vcm = =u
3m 3m (m + 2m)

Applying work energy theorem

∆K = WNet = Wsp + Wext + Wg + WN

At the position of maximum extension :

1 1 1  K 2
2
(3m) Vcm –  mu2 + (2m) (2u)2  = – X max + WFext + 0 + 0
2 2 2  2

3 mu2 m 2 –K 2
– u [1 + 8] = X max + Wext
2 2 2

mu2 −K 2
[3 – 9] = X max + Wext
2 2
2
– 6 mu2 −K Xmax
= + Wex ....(a)
2 2

Now the total work done by the external force.

Rankers Varanasi office | Pandeypur Chauraha – Call 8601467389


 F   F 
Wext = (F + ma)x1 + (2F – 2ma) x2 ⇒ Wext =  F + m  x1 +  2F − 2m x2
 3m   3m 

4F 4F 4F 4F
Wext = x1 + x2 = (x1 + x2) = xmax
3 3 3 3

Put in the eq. (a)

−6mu2 −Kx 2max 4F 8F 6mu2


= + Xmax ⇒ X2max – Xmax – =0
2 2 3 3K K

 +8F  64F2 6 mu2


 3K  ± + 4.
9 K2 K 4F ± 16F2 + 54 mu2 K
Xmax = –   ⇒ Xmax =
2 3K

m m
7. Sol. Note : velocity of the man is w.r.t. buggy given, so velocity of the man w.r.t.
m
ground is

→ → → → → →
VMB = VMG – VBG VMG = uMB + VBG


→ →
→ → → → – 2m u→
Case I Fext = 0 ⇒ P = constant vpj (M +2m). O = M V1 + 2 m ( u + V1 ) ⇒ V1 =
(2m + M)

Case II

→ → → →
(M + 2m) O = (m + M) V1 + m ( u + V1 )

→ →
Figure (A and B) – m u = (m + M + m) V1



–m u
⇒ V1 = Note buggy, move opposite to the man w.r.t. ground
(M + 2 m)

Figure (B and C)
→ →

–m u → → → –m u →
(m + M) = m (u + v 2 ) + M v 2 ⇒ (m + M) – mu = (m + M) v 2
(M + 2 m) (M + 2 m)

Rankers Varanasi office | Pandeypur Chauraha – Call 8601467389



[(m + M) + (M + 2m)](–m u ) →
⇒ = (m + M) v 2
(M + 2m)

→ →
→ (2M + 3m) (–m u ) –m(2M + 3m) u
v2 = =
(M + m)(M + 2m) (M + m)(M + 2m)


→ –m (2 M + 3m) u
V2 (m + M)(M + 2m)
2M + 3m 2 ( m + M) m
= = = +
→ → 2 (m + M) 2 (m + M) 2 (m + M)
V1 –2 m u
(M + 2m)

→ →
m
=1+ >1 (and t ⇒  V21  >  V11 
2 (m + M)

11. Sol. Let one ball strikes with another ball with velocity u.

To determine the x components of the final speed, apply the equations for conservation of
linear momentum.

m1 u1x + m2u2x = m1v1x + m2v2x

putting m1 = m2 = m

u1x + u2x = v1x + v2x

or u1x + 0 = v1x + v2x ...(i)

at the instant of maximum compression v1x = v2x = vx

(compression between them will increase till balls get same velocity in the x direction)

hence from equation (i)

v1x = v2x = vx

1
u cosα = vx + vx ⇒ vx = u cos α ...(ii)
2

 balls are smooth so 

Rankers Varanasi office | Pandeypur Chauraha – Call 8601467389


In the y direction ball 1 will continue with u sinα, perpendicular to the line of impact and at
the instant of maximum deformation loss of kinetic energy will be in the form of potential
energy

u
After collision velocity of ball 2, v2 = vx = cos α
2

u
After collision velocity of ball 1 = v1 = v 2x + v 2y (where vx = cos α)
2

2
u 
Hence v1 =  cos α  + (u sin α )2
2 

P.E. = Loss of K.E. of the system

1 1 1
∆ K.E. = m1 v12 + m2 v 22 – m1u2
2 2 2

 u 2  2
1  2 1 u  1
= m  cos α  + (u sin α )  + m  cos α  – mu2 { m=1 m
= 2 m}
2  2   2 2  2

2 2
1 u 1 1 u 1
∆ K.E. = m   cos2α + mu2 sin2 α + m   cos2 α – mu2 [sin2α + cos2α]
2 2 2 2 2 2

2
u 1 1
hence ∆ K.E. = m   cos2 α + mu2 sin2 α – mu2 [sin2 α + cos2 α]
2 2 2

u2 1
⇒ ∆ K.E. = m cos2 α – mu2 cos2 α
4 2

1 1
⇒ ∆ K.E. = mu2 cos2 α – mu2 cos2 α
4 2

1
⇒ ∆ K.E. = – mu2 cos2 α {– sign indicate loss of energy }
4

Fraction of energy in the from of P.E.

1
mu2 cos2 α
∆k PE 4 1 1 1
η= = = ⇒ η= cos2 α = (cos 45º)2 = = 0.25
ki ki 1 2 2 4
mu2
2

d
η η 4 − η2
16. Sol. sin θ = 2 = cos θ =
d 2 2

Rankers Varanasi office | Pandeypur Chauraha – Call 8601467389



Fext = 0 , so linear momentum of the system remain conserved.

 →
Px = constant, P y = constant

(along x-axis) ) mv = 2mvb cos θ – mv´

v = 2vb cos θ – v´ = vb 4 – η2 – v´ ....(i)

For elastic collision



| v rel−sepration |
e=  =1
| v rel approach |

⇒ v cos θ = vb + v´ cos θ

v 4 − η2 v + v´ v´ 4 − η2
v cos θ = = +
2 4 − η2 2

 4 − η2 2   2 4 − η2 
v −  = v´ + 
 1 4 − η2   4 − η2 1 
   

v(2 – η2) = v´ (6 – η2)

2 − η2
v´ = v
6 − η2

(If 2 > η2 then disc A recoil with v´)

Rankers Varanasi office | Pandeypur Chauraha – Call 8601467389


Rotational Motion
List of some useful formule :
Object Moment of Inertia

Solid Sphere

2
MR2 (Uniform)
5

Hollow Sphere

2
MR2 (Uniform)
3

Ring.

MR2 (Uniform or Non Uniform)

Disc

MR2
(Uniform)
2

Hollow cylinder

Rankers Varanasi office | Pandeypur Chauraha – Call 8601467389


MR2 (Uniform or Non Uniform)

Solid cylinder

MR2
(Uniform)
2

Thin rod

ML2
(Uniform)
3

ML2
(Uniform)
12

Two thin rod

2m2
(Uniform)
3

Rectangular Plate

M(a2 + b2 )
Ι= (Uniform)
12

Square Plate

Rankers Varanasi office | Pandeypur Chauraha – Call 8601467389


Ma2
ΙAB = ΙCD = ΙDF = (Uniform)
12

Square Plate

Ma2
(Uniform)
6

Cuboid

M(a2 + b2 )
(Uniform)
12

TORQUE:

Torque represents the capability of a force to produce change in the rotational motion of the
body.

6.1 Torque about a point:

 → → →
Torque of force F about a point τ = r × F

Where F = force applied

P = point of application of force

Q = Point about which we want to calculate the torque.



r = position vector of the point of application of force w.r.t. the point about which we
want to determine the torque.

Rankers Varanasi office | Pandeypur Chauraha – Call 8601467389



τ = r F sinθ = r⊥F = rF⊥

Where θ = angle between the direction of force and the position vector of P wrt. Q.

r⊥ = r sin θ = perpendicular distance of line of action of force from point Q, it is


also called force arm.
 
F⊥ = F sin θ = component of F perpendicular to r

SΙ unit of torque is Nm

 Torque is a vector quantity and its direction is determined using right hand thumb rule and its
always perpendicular to the plane of rotation of the body.

Torque about an axis :



The torque of a force F about an axis AB is defined as the component

of torque of F about any point O on the axis AB, along the axis AB.
→ → → →
In the given figure torque of F about O is τ0 = r × F

→ 
The torque of F about AB, τAB is component of τ0 along line AB.

There are four cases of torque of a force about an axis.:

 
Case I : Force is parallel to the axis of rotation, F || AB

AB is the axis of rotation about which torque is required


       
r × F is perpendicular to F , but F || AB , hence r × F is perpendicular to AB .
  
The component of r × F along AB is, therefore, zero.

Case II : The line of force intersects the axis of rotation (F intersect AB)

   
F intersects AB along r then F and r are along the same line. The torque about
O is
 
r × F = 0.

Hence component this torque along line AB is also zero.

Rankers Varanasi office | Pandeypur Chauraha – Call 8601467389


  
Case III : F perpendicular to AB but F and AB do not intersect.

In the three dimensions, two lines may be perpendicular without intersecting each
other.

Two nonparallel and nonintersecting lines are called skew lines.

Figure shows the plane through the point of application of force P that is
perpendicular to the axis of rotation AB. Suppose the plane intersects the axis at
the point O. The force F is in this plane (since F is perpendicular to AB). Taking the
origin at O,
   
Torque = r × F = OP × F .

Thus, torque = rF sin θ = F(OS)



where OS is the perpendicular from O to the line of action of the force F . The line
OS is also perpendicular to the axis of rotation. It is thus the length of the common
perpendicular to the force and the axis of rotation.
     
The direction of τ = OP × F is along the axis AB because AB ⊥ OP and AB ⊥
 
F . The torque about AB is, therefore, equal to the magnitude of τ that is F.(OS).

 Thus, the torque of F about AB = magnitude of the force F × length of the


common perpendicular to the force and the axis. The common perpendicular
OS is called the lever arm or moment arm of this torque.
 
Case IV : F and AB are skew but not perpendicular.

Here we resolve F into two components, one is parallel to axis and other is
perpendicular to axis. Torque of the parallel part is zero and that of the
perpendicular part may be found, by using the result of case (III).

Force Couple :
A pair of forces each of same magnitude and acting in opposite direction is called a force
couple. Torque due to couple = Magnitude of one force × distance between their lines of action.

Magnitude of torque = τ = F (2d)

Rankers Varanasi office | Pandeypur Chauraha – Call 8601467389


 A couple does not exert a net force on an object even though it exerts a torque.
 Net torque due to a force couple is same about any point.

Torque about A = x1F + x2F = F(x1 + x2) = Fd

Torque about B = y1F – y2F = F(y1 – y2) = Fd

 If net force acting on a system is zero, torque is same about any point.

 A consequence is that, if Fnet = 0 and τnet = 0 about one point, then τnet = 0 about any
point

Rotation about a fixed axis:


If ΙHinge = moment of inertia about the axis of rotation (since this axis passes through the hinge,
hence the name ΙHinge).

τext ) = resultant external torque acting on the body about axis of rotation

α = angular acceleration of the body.

 
τext )Hinge = ΙHinge α

Hinge x
1
Rotational Kinetic Energy = .Ι.ω2
2
Fixed axis of
  Rotation
P = Mv CM
 
Fexternal = M aCM

Net external force acting on the body has two component tangential and centripetal.

v2
⇒ FC = maC = m = mω2 rCM ⇒ Ft = mat = mα rCM
rCM

EQUILIBRIUM

Rankers Varanasi office | Pandeypur Chauraha – Call 8601467389


A system is in mechanical equilibrium if it is in translational as well as rotational equilibrium.

For this : Fnet = 0

τnet =0 (about every point)

Y
F1

F2
F5
X

F4 F3
 
From (6.3), if Fnet = 0 then τnet is same about every point
 
Hence necessary and sufficient condition for equilibrium is Fnet = 0 , τnet =0 about any one

point, which we can choose as per our convenience. ( τnet will automatically be zero about
every point)

unstable stable Neutral


equilibrium equilibrium equilibrium

The equilibrium of a body is called stable if the body tries to regain its equilibrium position
after being slightly displaced and released. It is called unstable if it gets further displaced
after being slightly displaced and released. If it can stay in equilibrium even after being
slightly displaced and released, it is said to be in neutral equilibrium.

ANGULAR MOMENTUM ( L )

8. 1. Angular momentum of a particle about a point.


  
L = r ×P ⇒ L = rpsinθ
 
or L = r⊥ × P or L = P⊥ × r


Where P = momentum of particle

r = position of vector of particle with respect to point O
about which angular momentum is to be calculated.
 
θ = angle between vectors r & P

r⊥ = perpendicular distance of line of motion of particle from point O.



P⊥= component of momentum perpendicular to r .

SI unit of angular momentum is kgm2/sec

Rankers Varanasi office | Pandeypur Chauraha – Call 8601467389


Conservation of Angular Momentum

 dL
Newton's 2nd law in rotation : τ =
dt
 
where τ and L are about the same axis.

 Angular momentum of a particle or a system remains constant if τext = 0 about the axis of
rotation.

Even if net angular momentum is not constant, one of its component of an angular
momentum about an axis remains constant if component of torque about that axis is zero

Impulse of Torque : ∫ τdt =∆J


∆J → Charge in angular momentum.

COMBINED TRANSLATIONAL AND ROTATIONAL MOTION OF A RIGID BODY :


The general motion of a rigid body can be thought of as a sum of two independent motions. A
translation of some point of the body plus a rotation about this point. A most convenient choice
of the point is the centre of mass of the body as it greatly simplifies the calculations.

 Consider a fan inside a train, and an observer A on the platform.

It the fan is switched off while the train moves, the motion of fan is pure translation as each
point on the fan undergoes same translation in any time interval.

It fan is switched on while the train is at rest the motion of fan is pure rotation about is axle
; as each point on the axle is at rest, while other points revolve about it with equal angular
velocity.

If the fan is switched on while the train is moving, the motion of fan to the observer on the
platform is neither pure translation nor pure rotation. This motion is an example of general
motion of a rigid body.

Now if there is an observer B inside the train, the motion of fan will appear to him as pure
rotation.

Hence we can see that the general motion of fan w.r.t. observer A can be resolved into
pure rotation of fan as observed by observer B plus pure translation of observer B (w.r.t.
observer A)

Such a resolution of general motion of a rigid body into pure rotation & pure translation is
not restricted to just the fan inside the train, but is possible for motion of any rigid system.

9.1 Kinematics of general motion of a rigid body :


For a rigid body as earlier stated value of angular displacement (θ), angular velocity (ω), angular
acceleration (α) is same for all points on the rigid body about any other point on the rigid body.

Hence if we know velocity of any one point (say A) on the rigid body and angular velocity of any
point on the rigid body about any other point on the rigid body (say ω), velocity of each point on
the rigid body can be calculated.

Rankers Varanasi office | Pandeypur Chauraha – Call 8601467389


 →
since distance AB is fixed VBA ⊥ AB VA
VB = VA+ VBA
VBA ⊥
we know that ω = B
rBA
V=
BA
ω × rAB
VBA⊥ = VBA = ωrBA VA
  
in vector form VBA = ω × rBA A
   rigid body
Now from relative velocity : VBA
= VB − VA
      
VB
= VA + VBA VB
⇒= VA +ω× r BA
   
similarly a=B a A + α × rBA [for any rigid system]

Example 1. Consider the general motion of a wheel (radius r) which can be view on pure translation of
its center O (with the velocity v) and pure rotation about O (with angular velocity ω)

ˆj
C 
ω = ω( −kˆ )

ˆi
D V B
O

A
       
Find out v AO ,vBO ,v CO ,vDO and v A , vB ,v C ,vD
  
Solution : v AO = ( ω × rAO )



( ˆ × OA
v AO = ω(–k) )

( ˆ × r( −ˆj)
v AO = ω(–k) )

v AO = −ωriˆ

similarly vBO = ωr ( −ˆj)
 ˆ ; 
v CO = ωr (i) vDO = ωr ( ˆj)
  
v A = v O + v AO= v ˆi − ωriˆ
  
Similarly vB = v O + vBO= v ˆi − ωrjˆ

  
v C = v O + v CO= v ˆi + ωriˆ
  
vD = v O + vDO= v ˆi + ωrjˆ

Rankers Varanasi office | Pandeypur Chauraha – Call 8601467389


Pure Rolling (or rolling without sliding) :
Pure rolling is a special case of general rotation of a rigid body with circular cross section (e.g.
wheel, disc, ring, sphere) moving on some surface. Here, there is no relative motion between
the rolling body and the surface of contact, at the point of contact


A

Here contact point is A & contact surface is horizontal ground. For pure rolling velocity of A
w.r.t. ground = 0 ⇒ VA = 0

 A
V

From above figure, for pure rolling, velocity of A w.r.t. to plank is zero ⇒ VA =V.

ω
r
v

A

From above figure for, pure rolling, velocity of A w.r.t. ground is zero.

⇒ v – ωr = 0

v = ωr

Similarly a = αr

Dynamics of general motion of a rigid body :


This motion can be viewed as translation of centre of mass and rotation about an axis passing
through centre of mass

If ΙCM = Moment of inertia about this axis passing through COM

τcm = Net torque about this axis passing through COM



aCM = Acceleration of COM

v CM = Velocity of COM

Fext = Net external force acting on the system.

Psystem = Linear momentum of system.

LCM = Angular momentum about centre of mass.

rCM = Position vector of COM w.r.t. point A.
 
then (i) τcm =Ιcm α

Rankers Varanasi office | Pandeypur Chauraha – Call 8601467389


 
(ii) Fext = Macm
 
(iii) Psystem = Mv cm

1 1
(vi) Total K.E.= Mv cm2 + Ιcm ω2
2 2
 
(v) LCM =ΙCMω
 
(vi) Angular momentum about point A = L about C.M. + L of C.M. about A
   
L A =Ιcm ω + rcm × Mv cm

dL A d    dω
 = ( Ιcm ω + rcm × Mv cm ) ≠ Ι A . Notice that torque equation can be
dt dt dt
applied to a rigid body in a general motion only and only about an axis through centre of mass

Instantaneous axis of rotation :


It is the axis about which the combined translational and rotational motion appears as pure
rotational motion.

The combined effect of translation of centre of mass and rotation about an axis through the
centre of mass is equivalent to a pure rotation with the same angular speed about a stationary
axis ; this axis is called instantaneous axis of rotation. It is defined for an instant and its position
changes with time.

eg. In pure rolling the point of contact with the surface is the instantaneous axis of rotation.

Geometrical construction of instantaneous axis of rotation (I.A.R). Draw velocity vector at


any two points on the rigid body. The I.A.R. is the point of intersection of the perpendicular
drawn on them.

 In case of pure rolling the lower point is instantaneously axis of rotation.


The motion of body in pure rolling can therefore by analysed as pure rotation about this
axis.

Consequently

τP = ΙPα

αP = ΙP ω

Rankers Varanasi office | Pandeypur Chauraha – Call 8601467389


K.E. = 1/2 ΙP ω2

Where ΙP is moment of inertial instantaneous axis of rotation passing through


P

The nature of friction in the following cases assume body is perfectly rigid

(i) v = ωR

ω
v

smooth surface

No friction and pure rolling.

(ii) v = ωR

ω
v

rough surface

No friction and pure rolling (If the body is not perfectly rigid, then there is a small friction
acting in this case which is called rolling friction)

(iii) v > ωR or v < ωR

ω
v

smooth surface

No friction force but not pure rolling.

(iv) v > ωR

ω
v
v>R
ω
fk
rough surface

There is Relative Motion at point of contact so Kinetic Friction, fk = µN will act in backward
direction. This kinetic friction decrease v and increase ω, so after some time v = ωR and
pure rolling will resume like in case (ii).

(v) v < ωR

Rankers Varanasi office | Pandeypur Chauraha – Call 8601467389


ω
v
v>R
ω
fk
rough surface

There is Relative Motion at point of contact so Kinetic Friction, fk = µN will act in forward
direction. This kinetic friction increase v and decrease ω, so after some time v = ωR and
pure rolling will resume like in case (ii).

(vi) v = ωR (initial)

ω
v F

smooth surface

No friction and no pure rolling.

(vii) v = ωR (initial)

ω
v F

fs
rough surface

Static friction whose value can be lie between zero and µsN will act in backward direction.
If coefficient of friction is sufficiently high, then fs compensates for increasing v due to F by
increasing ω and body may continue in pure rolling with increases v as well as ω.

Example 1. A cylinder is given angular velocity ω0 and kept on a horizontal rough surface
the initial velocity is zero. Find out distance travelled by the cylinder before it performs
pure rolling and work done by friction force

MR2 α ω0 v
Solution : µMg R = R
2
a v
R
2µg
α= .....(1) fk
R
S
Initial velocity u = 0

v2= u2 + 2as

v2 = 2as .....(2)

fK = Ma

µMg = Ma

a = µg .....(3)

Rankers Varanasi office | Pandeypur Chauraha – Call 8601467389


ω = ω0 – αt

2µg
from equation (1) ω = ω0 – t
R

v = u + at

from equation (3) v = µ g t

2v
ω = ω0 –
R

ω = ω0 – 2ω

ω0
ω=
3

from equation (2)

2
 ω0R 
  = (2as) = 2µ gs
 3 

 ω 2 R2 
s=  0 
 18 µg 
 

work done by the friction force w = (–fk R dθ + fk∆s)

µmg × ω02R2
– µmg R ∆θ +
18 µg

2
1 2 ω R 1 2µg  ω0R 
∆θ = ω0 × t – αt = ω0 ×  0  – ×  
2  3µg  2 R  3µg 

ω02R ω2R
– 0
3µg 9µg

2ω02R
9µg

2ω02R ω02R2
– µmg × R + µmg ×
9µg 18µg

2mω02R2 mω02R2
– +
9 18

−3mω02R2 mω02R2
=–
18 6

Alternative Solution

Rankers Varanasi office | Pandeypur Chauraha – Call 8601467389


Using work energy theorm w g + w a + w fk =
∆K

 1  ω R 2 1 mR2  ω 2   1 mR2   mω02R2 


w fk =  m  0  + × 0   −  × ω02  =  − 
 2  3  2 2  3    2 2  6 
  

Rolling on moving surface

Sufficiently Rough
m
a+R
α
 m F
/////////////////////////////////////////////

smooth

Friction on the plate backward or on cylinder friction forward so cylinder move forward.

m a
f

Because of pure rolling static friction f.


f
m F
mR2
fR = α
2 b

2f
α=
mR

f = ma

F – f = mb

F = m(a + b)

αR
a=
2

At contact point

b = a + αR

3α R
b=
2

b = 3a

F = 4ma

F
a=
4m

3F
b=
4m

Rankers Varanasi office | Pandeypur Chauraha – Call 8601467389


w.r.t. plate distance is covered = 
b a
and acceleration w.r.t. plate (b – a)

1 
= (b – a) t2
2

1 a× m
= × 2at2 = t = = 2
2 F F

TOPPLING :
In many situations an external force is applied to a body to cause it to slide along a surface. In
certain cases, the body may tip over before sliding ensues. This is known as topping.

(1) There is a no horizontal force so pressure at bottom is uniform and normal is colinear with
mg.

(2) If a force is applied at COM, pressure is not uniform Normal shifts right so that torque of N
can counter balance torque of friction.

Fmax = fr

N = mg

fr . b/2 = N . a/2 ⇒ fr = Na/b = mg a/b, Fmax = mg a/b

(3) If surface is not sufficiently rough and the body slides before F is increased to Fmax = mg
a/b then body will slide before toppling. Once body starts sliding friciton becomes constant
and hence no topping. This is the case if

Fmax > flimit

⇒ mg a/b > µmg

µ < a/b

Rankers Varanasi office | Pandeypur Chauraha – Call 8601467389


Condition for toppling when µ ≥ a/b in this case body will
topple if F > mg a/b

but if µ < a/b, body will not topple any value of F applied a
COM

Rankers Varanasi office | Pandeypur Chauraha – Call 8601467389


ADVANCED Level Question
1. Find the M.I. of a rod about (i) an axis perpendicular to the rod and passing through left end.
(ii) An axis through its centre of mass and perpendicular to the length whose linear density
varies as λ = ax where a is a positive constant and 'x' is the position of an element of the rod
relative to its left end. The length of the rod is .

a 4 a 4
Ans. (i) Ι = (ii)
4 36

Sol. (i)

dx

x
B

∫ dmx
2
ΙAB =


 a 4 
ΙAB = ∫ 0
ax 3 dx = 
 4



(ii)

dm

x dx

∫ ax dx
2

0 2 
xcm = Q
=  
3 
∫ ax dx
0

A ΙAB Ιcm

dx
2/3

B

2
2 
ΙAB = Ιcm + m   
3 

4m2
Ιcm = ΙAB –
9


a 2
m= ∫
0
ax dx =
2

Rankers Varanasi office | Pandeypur Chauraha – Call 8601467389


a 4 2a 4
Ιcm = −
4 9

 a 4 
Ιcm =  Ans.
 36 

7. A rotating disc (figure) moves in the positive direction of the x axis. Find the equation y (x)
describing the position of the instantaneous axis of rotation, if at the initial moment the axis C of
the disc was located at the point O after which it moved

(a) With a constant velocity v, while the disc started rotating counter clockwise with a
constant angular acceleration β (the initial angular velocity is equal to zero);

(b) With a constant acceleration a (and the zero initial velocity), while the disc rotates
counterclockwise with a constant angular velocity ω.

v2 2ax
Ans. (a) y = (Hyperbola) ; (b) y = (Parabola)]
βx ω

v2 2ax
Ans. (a) y = (vfrijyo;) ; (b) y = (ijoy;)]
βx ω

Sol. (a)

Let coordinates of instantaneous axis of rotation be P(x,y).

then velocity of P w.r.t. ground is zero.

 
⇒ ω × CP + v ˆi = 0

⇒ b t ( k̂ ) × [(x − v t) ˆi + yj]
ˆ + v ˆi =
0

⇒ x = vt

and βyt = V

Rankers Varanasi office | Pandeypur Chauraha – Call 8601467389


from these eliminating t

βy x v2
. =1 or xy =
v υ β

⇒ locus of P is a Hyperbola.

1 
(b) Here coordinate at point C =  Nt 2 , 0 
2 

1 
(b) C =  Nt 2 , 0 
2 
 
∴ ω × CP + v ˆi = 0

 1  1
⇒ ωkˆ × (x − w t 2 )iˆ + yjˆ  + w tiˆ =0 ⇒ x= w t2
 2  2

ωy = w t

from these eliminating t,


2
1 ω
⇒ x= w   y2
2 w

ω2 2
⇒ x= y
2 w

Eqn. of parabola.

8. A block X of mass 0.5 kg is held by a long massless string on a fixed frictionless inclined plane
inclined at 30º to the horizontal. The string is wound on a uniform solid cylindrical drum Y of
mass 2 kg and radius 0.2 m as shown in figure. The drum is given an initial angular velocity
such that block X starts moving up the plane.

(a) Find the tension in the string during motion.

(b) At a certain instant of time the magnitude of the angular velocity of Y is 10


rad s−1. Calculate the distance travelled by X from that instant of time until it comes to
rest.

Sol. a=αR

Rankers Varanasi office | Pandeypur Chauraha – Call 8601467389


mg sin 30 0 – T = ma .........(1)

mg
or – T = ma .........(2)
2

τ TR
α= =
I 1
MR2
2

2T
α= .........(3)
MR

Solving Equations (1), (2) and (3) for T, we get

1 M mg
T=
2 M + 2m

Substituting the value, we get

 1   (2)(0.5)(9.8) 
T=     = 1.63 N
 2   2 + (0.5)(2) 

T = 1.63 N

(ii) From Eq. (3) , angular retardation of drum

2T (2)(1.63)
α= = = 8.15 rad/s2
MR (2)(0.2)

or linear retardation of block

a = Rα = (0.2) (8.15) = 1.63 m/s2

At the moment when angular velocity of drum is

ω 0 = 10 rad/s

The linear velocity of block will be

v 0 = ω 0 R = (10) (0.2) = 2 m/s

Now, the distance (s) travelled by the block until it comes to rest will be given by

v 20
s= [ Using v 2 = v02 – 2as with v = 0 ]
2a

v 20
s= [ v 2 = v02 – 2as v = 0 ]
2a

(2)2
= m
2(1.63)

Rankers Varanasi office | Pandeypur Chauraha – Call 8601467389


or s = 1.22 m

Ans. (a) 1.633 N (b) 1.224 m [JEE


- 1994]

9. A rod of length R and mass M is free to rotate about a horizontal axis passing through hinge P
as in figure. First it is taken aside such that it becomes horizontal and then released. At the
lowest point the rod hits the small block B of mass m and stops. Find the ratio of masses such
that the block B completes the circular track of radius R. Neglect any friction.

P
m

Sol. Minimum velocity required by block ‘m’ to complete the motion in 5gR

conserving mech. energy

1 2 R MgR
Ιω = Mg . ⇒ ω=
2 2 Ι

Cons. angular momentum wrt P before & after collision.

Ιω = m.R 5gR

MgR
Ι = mR 5gR
Ι

MgRΙ = m2R2 5gR

ML2
putting Ι =
3

M
= 15
m

M
Ans. : = 15
m

10. A 3 kg uniform rod rotates in a vertical plane about a smooth pivot at B. A spring of constant k
= 300 N/m and of upstretched length 100 mm is attached to the rod as shown. Knowing that in
the position shown the rod has an angular velocity of 4 rad/s clockwise, determine the angular
velocity of the rod after it has rotated through. [g = 10 m/s2 ]

Rankers Varanasi office | Pandeypur Chauraha – Call 8601467389


(a) 90º

(b) 180º

2
Ans. (a) 86 rad/s (b) 4 rad/s
3

Sol. (i)

L = Natural length of spring

L = fLizax dks fcuk [kksyh voLFkk esa yEckbZ

(ii)

(a) By energy conservation from (i) to (ii)

(a)

1 2 1 2 1 2  
 Iω + kx = Iω1 + mg  − x   ............. (i)
2 2 2 2 

2
 
I = Icm +  − x  ,
2 

Rankers Varanasi office | Pandeypur Chauraha – Call 8601467389


2
m2  
I= + m  − x ............. (ii)
12  2 

(x = x2 + y2 − L ) ............. (iii)

Put equation (ii) and (iii) in equation (i)

1  m2  
( )
2 1/ 2
 1
 + m  − x   ω2 + K x2 + y2 − L
2  12 2   2

1  m2  
2
  
=  + m  − x   ω12 + mg  − x 
2  12  2    α 

x = 150 mm, y = 20 mm,  = 450 mm, K = 300 N/m

m = 3 kg, ω = 4 rad/sec

Put all the data

2
ω1 = 86 rad/sec
3

(b) rotating to 180° condition is

(b) 180° ?

This is like a initial condition so ω2 = ω

ω2 = 4 rad / sec

;
ω2 = 4 rad / sec.

   2
11. The angular momentum of a particle relative to a certain ' point O varies with time as M= a + bt
   
, where a and b are constant vectors, with a ⊥ b . Find the force moment N relative to the
point O acting on the particle when the angle between the vectors N and M equals 45°.
Irodov_1.185

a
Ans : N = 2b
b

Sol.

Rankers Varanasi office | Pandeypur Chauraha – Call 8601467389


Force moment relative to point O

 
 dM
N = = 2bt
dt
 
Let the angle between M and N

α = 45º at t = t0
    
1 M . N (a + bt 02 ).2bt 0
Then rc = =
2 M N a2 + b2 t 04 2bt 0

2b2 t 03 bt 02
= =
a2 + b2 t 4 . 2bt 0 a2 + b2 t 04

a
Solving, t0 = (as t0 cannot be negative)
b

   a
Therefore N 2bt
= = 0 2b
b

12. A plank of mass m1 with a uniform sphere of mass m2 placed on it rests on a smooth horizontal
plane. A constant horizontal force F is applied to the plank. With what accelerations will the
plank and the centre of the sphere move provided there is no sliding between the plank and the
sphere? Irodov_1.261

Ans : w1 = F/(m1 + 2/7m2); w2 = 2/7 w1.

Sol.

α = angular acceleration

α = angular acceleration

For the plank

F – ƒ = m1ω1 ....... (i)

For sphere torque about point C

Rankers Varanasi office | Pandeypur Chauraha – Call 8601467389


2
fr = Icα = m2r2α ....... (ii)
5

assuming ω2 is the acceleration of COM of sphere at point A

(ω1 = ω2 + αr) ....... (iii)

From equation (i), (ii) and (iii)

F 2 
ω1 = and ω2 =  ω1 
 2  7 
 m1 + 7 m2 
 

13. In the arrangement shown in the figure weight A possesses mass m, a pulley B possesses
mass M. Also known are the moment of inertia I of the pulley relative to its axis and the radii
of the pulley are R and 2R respectively. Consider the mass of the threads is negligible. Find
the acceleration of weight A after the system is set free. (Assume no slipping takes place
anywhere and axis of cylinder remains horizontal) Irodov_1.259

2R

R A

Ans : w = 3g (M + 3m) / (M + 9m + I/R2)

Sol.

T
T

2R
T1

T1
R A a1
a

For cylinder

Mg + T1 – 2T = Ma ......(i)

Torque about axis of rotation?

Rankers Varanasi office | Pandeypur Chauraha – Call 8601467389


 a
2TR + T1(2R) = Ι.α =  Ι . ....(ii) (a = αR)
 R 

For weight A

mg – T1 = ma1

No slipping between pulleys and thread

a1 = a + α(2R) = (3a) .... (iii)

From equation (i), (ii) and (iii)

 
 3(M + 3m) g 

a1 = 
  Ι 
 M + 9m + 
  R2  

14. A uniform rod AB of length  is released from rest with AB inclined at angle θ with horizontal. It
collides elastically with smooth horizontal surface after falling through a height h. What is the
height upto which the centre of mass of the rod rebounds after impact?

[RBD New Correction 2013_VA


Sir]

2
 1 − 3 cos2 θ  49 π 
Ans. H=  h ;h=
 1 + 3 cos2 θ  144
 

Sol. Velocity of end A at the moment it strikes ground = 2gh . If velocity of COM of rod just after
collision v′ and angular velocity acquired by the rod is ω clockwise as shown then using equation
for coefficient of restitution velocity of approach = velocity of sep. (applied at point A).

L
2gh = v′ + ω cosθ .............(1)
2

Angular momentum can be conserved about A just before collision & after collision as only
impulsive force will be acting at A only.

L L
2gh M cosθ = Ιcm ω – Mv′ cosθ .............(2)
2 2

2
Putting value of ω = ( 2gh – v′)
L cos θ

Rankers Varanasi office | Pandeypur Chauraha – Call 8601467389


from (1)

L ML2 2 L
2gh M. cosθ = ( 2gh – v′) – Mv′ cosθ
2 12 L cos θ 2

L L cos θv′ L 2gh 2gh


v′ + = Lcosθ
6 cos θ 2 6 cos θ 2

 1 + 3 cos2 θ  (1 − 3 cos2 θ)
v′   = 2gh
 6cos θ  6 cos θ

 1 − cos2 θ 
v′ =   2gh

 1 + 3 cos 

COM will of at maximum height when its velocity becomes zero during upward motion.

O = v′2 – 2g H
2
v′2  1 − 3 cos2 θ 
H= =  h.
2g  1 + 3 cos2 θ 

2
 1 − 3 cos2 θ  49 π 
[ Ans.: H =  h; h = ]
 1 + 3 cos2 θ  144
 

15. A uniform block A of mass 25 kg and length 6m is hinged at C and is supported by a small block
B as shown in the Figure . A constant force F of magnitude 400N is applied to block B
horizontally . What is the speed of B after it moves 1.5 m ? The mass of block B is 2.5 kg & the
coeffcient of friction for all contact surfaces is 0.3. [ use l n (3/2) 0.41 and g = 10 ms-2 ]

Ans. 323.4 m/s or 18.52 m/sec.

Sol. NC + NB = 250

NB – x = 250 × 3

750
NB =
x

750
f1 = µ
x

Rankers Varanasi office | Pandeypur Chauraha – Call 8601467389


 750 
f2 =  + 25  µ
 x 

work done against friction

4.5
 1500  3
W= ∫ ( ƒ1 + ƒ 2 ) dx = ∫
3
 x × 0.3 + 7.5  dx = 450 n 2 + 7.5 (4.5 – 3)
 

= 450 × 0.41 + 7.5 × 1.5

1
mv2 = 400 × 1.5 – 195.75
2

2
v2 = (600 – 195.75) × =161.7 × 2 = 323.4
2.5

v = 18.52 m/sec.

16. A window (of weight w) is supported by two strings passing over two smooth pulleys in the
frame of the window in which window just fits in, the other ends of the string being attached to
weights each equal to half the weight of the window. One thread breaks and the window moves
down. Find acceleration of the window if µ is the coefficient of friction, and 'a' is the height and
'b' the breadth of the window.

[S-02-03_Rotation_Ex-1_9]

 a − µb 
Ans. A=   g
 3a + µ b 

Sol. θ is very small

θ ≈ 0º

T
b µ N1
N1 P

T a
µ N2
θ A
A N2

w/2

Rankers Varanasi office | Pandeypur Chauraha – Call 8601467389


Force balance in horizontal direction

N1 = N 2

balancing torque about point P

For θ to be very small we can directly write

Wb
T.b + µN2b – – N2a = 0
2

Force in y direction if acceleration of windows is A

wA
w – µN1 – µN2 – T = ... (ii)
g

For block

W WA
T– =
2 2g

 W WA 
T=  +  .... (iii)
 2 2g 

Put equation (iii) in equation (i)

W W Wb
b+ Ab + µN1b = N1a +
2 2g 2

WAb
= N1 (a – µb)
2g

 WAb 
N1 =   ..... (iv)
 2g (a − µb) 

Put N1 and T in equation (ii)

 WAb  W WA WA
W – 2µ   – – =
 2g (a − µb)  2 2g g

W µ WAb 3WA
– =
2 g (a − µb) 2g

2µ Ab 3A
1– =
g (a − µb) g

g (a – µb) = (2µb + 3a – 3µb)A

(a − µb) g
A= Ans.
(3a − µb)

Rankers Varanasi office | Pandeypur Chauraha – Call 8601467389


17. Three particles A, B, C of mass m each are joined to each other by massless rigid rods to form
an equilateral triangle of side a. Another particle of mass m hits B with a velocity v0 directed
along BC as shown. The colliding particle stops immediately after impact.

(a) Calculate the time required by the triangle ABC to complete half revolution in its subsequent
motion.

(b) What is the net displacement of point B during this interval?

6a π a
(2 π )
2
Ans. (a) t = (b) s = 1 + + 3
3 ν0 3

Sol. After collision, let COM move by velocity v′ and

system starts rotating by angular velocity ω about COM.

Using cons. of linear momentum

v0
mv0 = 3mv′ ⇒ v′ =
3

conserving angular momentum about COM

a  ma2 
mv0. = Ιω =  × 3  .ω
2 3  3 
 

= ma2ω

v0
ω=
2 3 a

(a) Time to complete half revolution.

π 2 3 aπ
t= =
ω v0

(b) Particle ‘B’ completes half cycle during this duration. It’s position const. COM in
shown.

Rankers Varanasi office | Pandeypur Chauraha – Call 8601467389


Disp. of B in x-direction = Disp. due to linear motion of COM

+ Disp. due to Angular motion.

v0
xB = .t + MN
3

v0 2 3 aπ 2a 2
= . + . cos30° = aπ + a
3 v0 3 3

Disp. in Y-direction

2a a
YB = cos60° =
3 3

Total displacement = xB2 + yB2

18. Disk A has a mass of 4 kg and a radius r = 75 mm, it is at rest when it is placed in contact
with the belt, which moves at a constant speed υ = 18 m/s . Knowing that µk = 0.25 between
the disk and the belt, determine the number of revolutions executed by the disk before it
reaches a constant angular velocity . (Assume that the normal reaction by the belt on the disc
is equal to weight of the disc) .

216
Ans.
π

Sol.

Rankers Varanasi office | Pandeypur Chauraha – Call 8601467389


ω

f = µ mg

Torque about A

mR2
R(µ mg) = ⋅α
2

2µg
= α
R

2 × 0.25 × 10
= α
R

5
α=  
R

at constant angular speed

v
ω=  
R

2
v
2 (2πn)
 R  =α
 

 v2  18 × 18
n=   =
 4απR2
  4 × 5.R.π

 18 × 18  18 × 18
n=   =
 20.R.π  20 × 75 × 10−3 × π

18 × 18 × 103  6 × 18 × 103 
n= =  
20 × 75 × π  20 × π × 20
 

 6 × 18 × 4 
n=  
 π×2 

36π6 216
n= =
π π

Number of revolutions executed by the disk before it comes at constant angular velocity n =
 216 
 .
 π 

Rankers Varanasi office | Pandeypur Chauraha – Call 8601467389


19. A 160 mm diameter pipe of mass 6 kg rests on a 1.5 kg plate . The pipe and plate are
initially at rest when a force P of magnitude 25 N is applied for 0.75 s . Knowing that µs =
0.25 & µk = 0.20 between the plate and both the pipe and the floor, determine ;

(a) whether the pipe slides with respect to the plate.

(b) the resulting velocities of the pipe and of the plate.

5 125 5
Ans. (a) pipe rolls without sliding (b) pipe : m/s → , rad/s (ccw) ; plate : m/s →
6 24 3

Sol.

α
a2
m
f2

Friction on plate due to ground f1 = 7.5 × 0.2 × 10 = 15

25 – 15 – f2 = 1.5 a1

f2 = 6a2

10 = 1.5 a1 + 6a2 ....(i)

f2 . r = mr2 . α

⇒ f2 = ma2 ...(ii)

f2 = ma1 – ma2

a2 + rα = a1 ⇒ a1 – a2 = a2

⇒ a2 = a1 – a2

⇒ a1 = 2a2

10 – 3a1 = 1.5 a1

100 20
⇒ a1 = =
45 9

a1 20
a2 = =
2 18

20 3 5
v1 = a1t = × = (Plate)
9 4 3

20 3 5
v2 = a2t = × = (pipe)
18 4 6

Rankers Varanasi office | Pandeypur Chauraha – Call 8601467389


v 2 5 1000
ω
=2 = =x 10.42 rad / s (pipe)
r 6 160
2

20. A uniform disc of mass m and radius R is rolling up a rough inclined plane, which makes an
angle of 30º with the horizontal. If the coefficients of static and kinetic friction are each equal to
µ and only the forces acting are gravitational, normal reaction and friction, then the magnitude
of the frictional force acting on the disc is ______ and its direction is _____ (write 'up' or 'down')
the inclined plane.[JEE - 1997]

mg
Ans. , up
6

Sol. Under the given conditions only possibility is that friction is upwards and it accelerates
downwards as shown below :

The equations of motion are :

mg sin θ – f mg sin 30º – f g f


a= = = – ......(1)
m m 2 m

τ fR 2f
α= = = ......(2)
Ι Ι mR

For rolling (no slipping)

a = Rα or g/2 – f/m = 2f/m

3f
∴ = g/2 or f = mg/6
m

(1) f ≤ fmax

3
≤ µ mg cos 30º ≤ µ mg
2

(2) Other possibilities which are not feasible are as follows :

(a) Friction is downwards.

In this case a and α will be as shown and rolling is not possible.

(b) Friction is upwards and the disc has linear acceleration in upward direction.

Rankers Varanasi office | Pandeypur Chauraha – Call 8601467389


In this case also rolling is not possible.

21. A uniform disc of mass m and radius R is projected horizontally with velocity ν0 on a rough
horizontal floor so that it starts off with a purely sliding motion at t = 0. After t0 seconds, it
acquires a purely rolling motion as shown in figure.

V0

///////////////////// /////////////////////
t=0 t = t0

(a) Calculate the velocity of the centre of mass of the disc at t0 .

(b) Assuming the coefficient of friction to be µ, calculate t0. Also calculate the work done by the
frictional force as a function of time & the total work done by it over a time t much longer than
t0. [JEE - 1997]

2 ν0 ν 3 1
Ans. (a) ν = ; t0 = 0 (b) w = –µmg (v0 t – µgt2) ; – mν02
3 3µg 2 6

Sol. Between the time t = 0 to t = t0. There is forward sliding, so friction, f is leftwards and
maximum i.e., µ mg. For time t > t0, friction f will become zero, because now pure rolling has
started i.e., there is no sliding (no relative motion) between the points of contact.

So, for time t < t0

f
Linear retardation, α = = µg (f = µ mg)
m

f R
and angular acceleration, α =
τ
= I mR2 = 2µg
I 2 R

Now let V be the linear velocity and ω, the angular velocity of the disc at time t = t0 then

V = V0 – at0 = V0 – µgt0 ......(1)

Rankers Varanasi office | Pandeypur Chauraha – Call 8601467389


2µgt 0
and ω = α t0 = ......(2)
R

For pure rolling to take place

V = Rω

i.e., V0 –2µto = 2µto

V0
⇒ t0 =
3 µ g

Substituting in Eq. (1), we have

 V0 
V = V0 – µg  
3 µ g

2
V= V0
3

Work done by friction

For t ≤ t0, linear velocity of disc at any time t is V = V0 – µgt and angular velocity is ω = αt =
2µgt
. From work-energy theorem, work done by friction upto time t = Kinetic energy of the
R
disc at time t – Kinetic energy of the disc at time t = 0

1 1 1
∴ W= mV2 + Iω2 – mV02
2 2 2
2
1 1 1   2µgt  1
= m [V0 – µgt]2 + mR2    – mV02
2 2  2  2  2

= [mV02 + mµ2g2t2 – 2mV0µgt + 2mµ2g2t2 – mV02]

mµgt
or W= [3µgt – 2V0]
2

For t > to, friction force is zero i.e., work done by friction is zero. Hence, the energy will be
conserved.

Therefore, total work done by friction over a time t much longer then t0 is total work done upto
time t0 (because beyond the work done by friction is zero) which is equal to

mµgt 0
W= [3µgt0 – 2Vo]
2

Substituting t0 = V0/3µg, we get

mV0
W= [V0 – 2V0]
6

mV02
W=–
6

22. A carpet of mass 'M' made of inextensible material is rolled along its length in the form of a
cylinder of radius 'R' and is kept on a rough floor. The carpet starts unrolling without sliding on
the floor when a negligibly small push is given to it. Calculate the horizontal velocity of the axis

Rankers Varanasi office | Pandeypur Chauraha – Call 8601467389


of the cylindrical part of the carpet when its radius reduces to R/2.
[JEE - 1990]

R
R/2

//////////////////////////////// ////////////////////////////////

14 g R
Ans. υ=
3

Sol. Let M be the mass of unwound carpet. Then ,

2
 M  R M
M‘ =  2 
π   =
 πR  2 4

M
M’

v
R
R/2

From conservation of mechanical energy :

R 1 M 2 1
MgR – M ‘ g = v + I ω2
2 2  4  2

 M   R  Mv 2 1  1 M R2   v 2
or ; MgR –   g   = +  × ×   
4 2 8 2  2 4 4   R/2 

7 3Mv 2 14 Rg
or ; MgR = ∴ v=
8 16 3

23. Figure shows a vertical force F that is applied tangentially to a uniform cylinder of weight W.
The coefficient of static friction between the cylinder and all surfaces is 0.5. Find in terms of W,
the maximum force that can be applied without causing the cylinder to rotate.

Rankers Varanasi office | Pandeypur Chauraha – Call 8601467389


3W
Ans.
8

Sol. When F is maximum equation. of rotational equilibrium.

F.R. = µ (N1 + N2) R .............(1)

For equilibrium in horizontal direction

f1 = N2 = µN1 ............(2)

In vertical direction

F + N1 = mg

F = µ [(mg – F) + µ (mg – F)]

1  1   1  1 
2 (mg − F) + 2 (mg − F) putting µ = 2  µ = 2 j [ kusi j 
    

 1 1 3
F 1 + +  = mg
 2 2 4

3 3
F= mg = w
8 8

[ Ans.: 3w/8 ]

24. A drinking straw of mass 2m is placed on a smooth table orthogonally to the edge such that
half of it extends beyond the table. A fly of mass m lands on the A end of the straw and walks
along the straw untill it reaches the B end. It does not tip even when another fly gently lands on
the top of the first one. Find the largest mass that the second fly can have. (Neglect the friction
between straw and table).

Ans. 3m

Sol. As fly moves to other end C.M must remains at same position so straw shifts left.

Rankers Varanasi office | Pandeypur Chauraha – Call 8601467389


Torque about AB is balanced AB

 
2mg   = (m + mA)g  
3 6

4m = m + mA

mA = 3m

25. Two uniform thin rods A & B of length 0.6 m each and of masses 0.01 kg & 0.02 kg respectively
are rigidly joined, end to end. The combination is pivoted at the lighter end P as shown in figure
such that it can freely rotate about the point P in a vertical plane. A small object of mass 0.05
kg moving horizontally hits the lower end of the combination and sticks to it. What should be
the velocity of the object so that the system could just be raised to the horizontal position ?
[JEE - 1994]

P
x

B
m v

Ans. 6.3 m/s

Sol. System is free to rotate but not free to translate. During collision, net torque of the system ( rod
A + rod B + mass m ) about point P is zero.

Therefore, angular momentum of system before collision = Angular momentum of system just
after collision. ( About P ). Let ω be the angular velocity of system just after collision, then

B
v
m

Li = Lf

⇒ mv (2l) = ω

Here,  = moment of inertia of system about P

Rankers Varanasi office | Pandeypur Chauraha – Call 8601467389


 2   2
= m (2) 2 + m A ( 2 / 3 ) + m B  +  +  )} 
 12  2 

Given:  = 0.6 m, m = 0.05 kg, mA = 0.01 kg and mB = 0.02 kg

Substituting the values, we get

I = 0.09 kg–m 2

Therefore, from Eq. (1)

2mv (2)(0.05)(v)(0.6)
ω= =
I 0.09

ω = 0.67 v ........(2)

Now after collision, mechanical energy will be conserved.

-0

Therefore, decrease in rotational KE = increase in gravitational PE

1 2   
or ; Iω = mg (2) + mA g   + m B g  + 2 
2 2  

g(4m + mA + 3 mB )
or ; ω2 =
I

(9.8) ( 0.6 ) (4 × 0.05 + 0.01 + 3 × 0.02)


=
0.09

= 17.64 (rad /s) 2

∴ ω = 4.2 rad/s .........(3)

Equating Eqs. (2) and (3) , we get

4.2
v= m/s
0.67

or ; v = 6.3 m/s

26. A uniform cube of side 'a' and mass m rests on a rough horizontal table. A horizontal force F is
applied normal to one of the faces at a point directly above the centre of the face, at a height

Rankers Varanasi office | Pandeypur Chauraha – Call 8601467389


3a
above the base. (i) What is the minimum value of F for which the cube begins to tip about
4
an edge?

(ii) What is the minimum value of µs so that toppling occures.

(iii) If µ = µmin , find minimum force for topping.

(iv) Minimum µs so that Fmin (as in part-(i)) can cause toppling.

2 2
Ans. (i) mg, (ii) µmin = 0, (iii) F = 2 mg], (iv) µs =
3 3

Solution :

(i) In the limiting case normal reaction will pass through O. The cube will tip about
O if torque of Fabout O exceeds the torque of mg.

 3a  a
Hence F   > mg  
 4  2

N
F
a/2 3a/4
fr
O
mg

2
or ;k F> mg
3

2
therefore, minimum value of F is mg.
3

(ii) In this case since it is not acting at COM, toppling can occur even after body
started sliding because of increasing the the torque of F about COM.hence µmin
= 0,

(iii) Now body is sliding before toppling, O is not I.A.R., torque equation can not be
applied across it. It can now be applied about COM.

(iii)

a a
F× =N× ................ (1)
4 2

N = mg .......................... (2)

from (1) and (2) lHkh (1) vkSj (2) ls


F = 2 mg

2
(iv) F> mg ................... (1) (from sol. (i))
3

Rankers Varanasi office | Pandeypur Chauraha – Call 8601467389


N = mg .......................(2)

F = µsN = µsmg ........... (3) from (1) and (2)

2
µs =
3

27. Three particles A , B and C each of mass m are connected to each other by three massless
rigid rods to form a rigid , equilateral triangular body of side  . This body is placed on a
horizontal frictionless table (x - y plane) and is hinged to it at the point A so that it can move
without friction about the vertical axis through A as shown in figure. The body is set into
rotational motion on the table about A with a constant angular velocity ω

(a) Find the magnitude of the horizontal force exerted by the hinge on the body .

(b) At time T , when the side BC is parallel to the x − axis , a force F is applied on B along
BC as shown. Obtain the x − component and the y − component of the force exerted by
the hinge on the body, immediately after time T.
[JEE Mains 02, (1+4)/60]

Ans. : (a) 3 m ω2  (b) Fy = 3 m ω2  Fx = − F/4 ]

Sol. (a) The distance of centre of mass (COM) of the system about point A will be :


r=
3

Therefore the magnitude of horizontal force exerted by the hinge on the body is

F = centripetal force

or F = (3m) rω2

   2
or F = (3’m)  ω
 3

or F= 3 mω2 Ans.

(b) Angular acceleration of system about point A is

τA
α=
IA

Rankers Varanasi office | Pandeypur Chauraha – Call 8601467389


 3 
(F) 
 2  
=  
2
2m
3 F
=
4m

Now acceleration of COM along x-axis is

y
A
ωα
,

x
 
3/2 

COM
F
B  C

   3 
αX = rα =    
 3   4 m 

F
or ax =
4m

Now let Fx be the force applied by the hinge along x-axis. Then :

Fx + F = (3m) ax

 F 
or Fx + F = (3m)  
 4m 

3 F
or Fx + F = F or Fx = – Ans.
4 4

Further if Fy be the force applied by the hinge along y-axis. Then :

Fy = centripetal force

or Fy = 3 mω2 Ans.

28. A bar of mass m is held as shown between 4 disks , each of mass M & radius r = 75
mm Determine the acceleration of the bar immediately after it has been released from rest,
knowing that the normal forces exerted on the disks are sufficient to prevent any slipping and
assuming that ;

In (i) case the discs are attacthed to the fixed support on wall. In (ii) case the discs are attached
to the bar.

Rankers Varanasi office | Pandeypur Chauraha – Call 8601467389


(a) m = 5 kg and M = 2 kg .

(b) the mass of M of the disks is negligible.

(c) the mass of m of the bar is negligible .

13 g 2g
Ans. (i) (a) 5g/9 ↓ (b) g ↓ (c) 0 (ii) (a) ↓ (b) g ↓ (c) ↓
17 3

Sol. (i)

M M

M f M

(a) mg – 4f = ma ......... (i)

Ιa
fR = Ια =
R

fR2 = Ιa

MR2
fR2 = a
2

 Ma 
f=  
 2 

 4Ma 
mg =  + ma  = (2M + m)a M = 2kg, m = 5 kg
 2 

5g
a= (↓)
9

(b) If ;fn M=0 (c) m=0

f=0 mg = (2M + m) a

Rankers Varanasi office | Pandeypur Chauraha – Call 8601467389


mg = ma 0=a

a = g(↓) a=0

(ii)

(a) (m + 4M)g – 4f = (m + 4M)a

Torque about centre of disk (α = a / R)

MR2 a
f.R= .
2 R

 Ma 
f=  
 2 

(m + 4M) g – 2Ma = (m + 4M) a

(m + 4M) g = (m + 6M) a

(5 + 8) g = (5 + 12) a

 13g 
a=   (↓)
 17 

(b) If ;fn M=0 If ;fn M=0

mg = ma 4Mg = 6Ma

2g
a = g(↓) a= (↓)
3

29. Two thin circular discs of mass 2 kg and radius 10 cm each are joined by a rigid massless rod
of length 20 cm. The axis of the rod is along the perpendicular to the planes of the disk through
their centres. This object is kept on a truck in such a way that the axis of the object is horizontal
and perpendicular to the direction of motion of the truck. The friction with the floor of the truck
is large enough, so that object can roll on the truck without slipping. Take x-axis as the direction
of motion of the truck and z-axis as the vertically upward direction. If the truck has an
acceleration of 9 m/s2, calculate [JEE - 1997’ 5/100]

Rankers Varanasi office | Pandeypur Chauraha – Call 8601467389


(a) the force of friction on each disc.

(b) the magnitude & direction of frictional torque acting on each disk about the centre of mass
O of the object. Express the torque in the vector form in terms of unit vectors ˆi , ˆj & kˆ along
x, y & z direction.
 
Ans. τ1 0.6kˆ − 0.6 ˆj , τ2 =− 0.6kˆ − 0.6 ˆj
(a) 6 N (b)=

Sol. Given mass of disc m = 2Kg and radius R = 0.1 m

(i) FBD of any one disc is

z
Truck a = 9m/s
2

×
y
x

Frictional force on the should be in forward direction.

α
a0

f
P

Let a0 be the acceleration of COM of disc α the angular acceleration about its COM. Then –

f
Q
a = 9m/s
2

f f
a0 = = ......(i)
m 2

τ f. R 2f 2f
α= = = = = 10 f .......(2)
I 1 mR 2 × 0.1
mR2
2

Since there is no slipping between disc and truk therfore.

Acceleration of point P = Acceleration of point Q

∴ a0 + Rα =a

f
or ;k  2  + (0.1)(10 f) =
 

Rankers Varanasi office | Pandeypur Chauraha – Call 8601467389


3 2a 2 × 9.0
or ;k f=a f= = N
2 3 3

∴ f = 6N

Since this force is acting in positive x-direction.

Therefore, in vector form



f =( 6 î ) N Ans. 3 (i)

→ → →
(ii) τ = r × f


Here f = (6 î ) N ( for both the discs

f = (6 î ) N

→ →
rP r1
= = 0.1 ĵ – 0. 1 k̂ and vkSj

→ →
rQ r2
= = 0.1 ĵ – 0. 1 k̂ and vkSj

20cm = 0.2 m

1 2
z
O
y
x

P Q
f f

Therefore, frictional torque on disk 1 about O (centre of mass )

→ → →
τ = r × f = ( –0.1 ĵ – 0.1 k̂ ) × (6 î ) N-m

= ( 0.6 k̂ – 0.6 ĵ )


r1
or ;k = 0.6 ( k̂ – ĵ ) N-m ⇒ 0.6 ( k̂ – ĵ )


r1
and | | = (0.6)2 + (0.6)2 = 0.85 N-m

→ → →
r1 r2
Similarly, = × f = ( 0.1 ĵ – 0.1 k̂ ) × (6 î ) N-m


r1
= 0.6 ( – ĵ – k̂ ) 0.6 ⇒ ( k̂ – ĵ )

Rankers Varanasi office | Pandeypur Chauraha – Call 8601467389


→ →
r2 r1
and | | =| | = 0.85 N-m Ans. 3 (ii)

30. A rectangular rigid fixed block has a long horizontal edge. A solid homogeneous cylinder of
radius r is placed horizontally at rest with its length parallel to the edge such that the axis of the
cylinder and the edge of the block are in the same vertical plane. There is sufficient friction
present at the edge so that a very small displacement causes the cylinder to roll off the edge
without slipping. Determine :

(a) The angle θc through which the cylinder rotates before it leaves contact with the edge.

(b) The speed of the centre of mass of the cylinder before leaving contact with the edge.

(c) The ratio of translational to rotational kinetic energies of the cylinder when its centre of mass
is in horizontal line with the edge. [JEE - 1995]

[JEE - 1995]

4 4 k
Ans. (a) θ = cos-1 (b) v = gr (c) T = 6
7 7 kR

Sol. (a) The cylinder rotates about the point of contact. Hence, the machanical energy of the cylinder
will be conserved i.e.,

ω
θ
R os
θ Rc

∴ (PE + KE ) 1 = ( PE + KE ) 2

1 2 1
∴ mgr + 0 = mgr cosθ + Iω + mv 2
2 2

V’

Rankers Varanasi office | Pandeypur Chauraha – Call 8601467389


but ω = v / R ( No slipping at point of contact. )

ysfdu ω = v / R

1
and vkSj I= mv 2
2

Therefore, vr%

1 1   v2  1
mgR = mgR cosθ +  mR2   2  + mv 2
2 2   R  2

3 2
or ;k v = gR ( 1 – cosθ )
4

4
or ;k v2 = gR ( 1 – cosθ )
3

v2 4
or ;k = gR ( 1 – cosθ ) .............(1)
R 3

s
co V
mg

N=0
mg

At the time of leaving contact, normal reaction N = 0 and θ = θ c hence,

mv 2
mg cosθ =
R

v2
or ;k = g cosθ ...........(2)
R

From Eqs. (1) and (2)

4
g ( 1 – cosθ c ) = g cosθ c
3

7
or ;k cosθ c = 1
4

or ;k cosθ c = 4 / 7

or ;k θ c = cos – 1 ( 4 / 7 )

4
(b) v= gR(1 − cos θ) [From Eq. (1)] [lehdj.k (1) ls ]
3

At the time of losing contact

cosθ = cosθ c = 4 / 7

Rankers Varanasi office | Pandeypur Chauraha – Call 8601467389


4  4
∴ v= gR  1 − 
3  7 

4
v= gR
7

4
Therefore, speed of COM of cylinder just before losing contact is gR
7

1
Therefore, rotational kinetic energy K R = Iω 2
2

1 1  v2
or ;k KR =  mR 2  2
2 2  R

1
= mv 2
4

1 4 
= m  gR 
4 7 

mgR
or ;k KR =
7

Now, once the cylinder losses its contact, N = 0, i.e., the frictional force , which is responsible
for its rotation, also vanishes. Hence, its rotational kinetic energy now becomes constant, while
its translational kinetic energy increases.

Applying conservation

decrease in gravitational PE = Gain in rotational KE + translational KE

∴ Translational KE (K T) = Decrease in gravitational PE – K R

mgR 6
or ;k KT = (mgR) – = mgR
7 7

From Eqs. (3) and (4)

6
mgR
KT 7
=
KR mgR
7

KT
or ;k =6
KR

31. A wedge of mass ‘m’ and triangular cross section (AB = BC = CA = 2R) is moving with a
constant velocity-v î towards a sphere of radius R fixed on a smooth horizontal table as shown
in the figure. The wedge makes an elastic collision with the fixed sphere and returns along the
same path without any rotation. Neglect all friction and suppose that the wedge remains in
contact with the sphere for a very short time ∆t, during which the sphere exerts a constant force

F on the wedge. The sphere is always fixed.

Rankers Varanasi office | Pandeypur Chauraha – Call 8601467389


 
(a) Find the force F and also the normal force N exerted by the table on the wedge during the
time ∆t.

(b) Let ‘h’ denote the perpendicular distance between the centre of mass of the wedge and the
 
line of action of force F . Find the magnitude of the torque due to the normal force N about the
centre of the wedge, during the time ∆t. [JEE - 1998, 8]

 2mV 2mV ˆ   2mV    4mVh 


Ans. (a) F = ˆi − k; N=
 + mg  kˆ , (b) τ = −   ĵ
∆t 3∆t  3∆t   3∆t 

Sol. (a)

-v ˆi
v ˆi

Fcos30º
z 30º
y
Fixed F
x Fsin30º

(i) Since the collision is elastic, the wedge will return with velocity v .

Now -

Linear impulse in x-direction = change in momentum in x - direction.

∴ (F cos 30º )∆t = mv – (mv ) = 2 mv

2mv 4mv
∴ F= =
∆t cos30º 3 ∆t

4mv
∴ F=
3 ∆t

∴ F = F cos 30º î – ( F sin 30º) k̂

  2mv   2mv 
or F =   î –   k̂ Ans. 3a (i)
 ∆t   3 ∆t 

(ii) Taking the equilibrium of wedge in vertical

(z) direction during collision

N = mg + F sin 30º

2mv
N = mg +
3 ∆t

Rankers Varanasi office | Pandeypur Chauraha – Call 8601467389


N

c
mg

Fsin 30º

or in vector form

  2mv 
N =  mg +  k̂ Ans. 3a (ii)
 3 ∆t 

(b) For rotational equilibrium of wedge ( about COM )

anticlockwise torque of F

= clockwise torque due to N

N C
h
mg

∴ magnitude of torque of N about COM = magnitude of torque of F about COM

= F.h

  4mv  ∧
τN = −  j Ans. 3 (b)
 3 ∆t 

Sol. (a)

-v ˆi
v ˆi

Fcos30º
z 30º
y
Fixed F
x Fsin30º

(i)

∴ (F cos 30º )∆t = mv – (mv ) = 2 mv

2mv 4mv
∴ F= =
∆t cos30º 3 ∆t

Rankers Varanasi office | Pandeypur Chauraha – Call 8601467389


4mv
∴ F=
3 ∆t

∴ F = F cos 30º î – ( F sin 30º) k̂

  2mv   2mv 
;k F =   î –   k̂ Ans. 3a (i)
 ∆t   3 ∆t 

(ii)

N = mg + F sin 30º

2mv
N = mg +
3 ∆t

c
mg

Fsin 30º

;k lfn'k :i esa
  2mv 
N =  mg +  k̂ Ans. 3a (ii)
 3 ∆t 

(b)

N C
h
mg

= F.h

  4mv 
τN =   k̂ Ans. 3 (b)
 3 ∆t 

32. The surface mass density (mass/area) of a circular disc of radius 'R' depends on the distance
from the centre x given as, σ(x) = α + βx. Where α and β are positive constant find its moment
of inertia about the line perpendicular to the plane of the disc through its centre.From Q.4
Exercise # 2 Part - I

 αR 4 β R5 
Ans. 2π  + 
 4 5 

Sol.

Rankers Varanasi office | Pandeypur Chauraha – Call 8601467389


dm = (2πxdx)σ
R

∫ dm · x ∫ (2πxdx)σ.x
2 2
Ι= =
o


Ι = 2π x 3 σdx
0

R R R

∫ ∫α ∫
3
Ι = 2π x ·(α + βx)dx = 2π  x dx + β x 4 dx
3

0 o o

 αR 4 β R 5 
Ι = 2π  + 
 4 5 

33. Calculate the moment of inertia of a uniform solid cone relative to its symmetry axis, if the mass
of the cone is equal to m and the radius of its base to R. From Q.5 Exercise # 2 Part –
I
Ans. Ι = 3/10 mR2

Sol.

h y
=
R r

R
r= y
h

dm = ρ (πr2dy)

1
dΙAB = (dm) r2
2

Rankers Varanasi office | Pandeypur Chauraha – Call 8601467389


h

∫ 2 (ρπr dy )
1 2
ΙAB = r2
y =0

ρπ R 4  h5 
=  
2 h4  5 

   
π 4  m   m 
= R h.   ...........  ρ = 
10  1 πR2h   1 2 
πR h 
 3   3 

3
= mR2
10


34. A force F = A î + B ĵ is applied to a point whose radius vector relative to the origin of

coordinates O is equal to r = a î + b ĵ , where a, b &, A, B are constants, and î , ĵ are the

unit vectors of the x and y axes. Find the moment N ( torque of F ) and the arm  of the force
relative to the point O.

From Q.6 Exercise # 2 Part - I Irodov_1.235

2 2
[Ans : N = (aB – bA) k̂ , where k̂ is the unit vector of the z axis  = |aB –bA|/ A + B ]
Irodov_1.235

 
Sol. N = r × F = (a ˆi + bj)
 × (Aiˆ + Bj)
ˆ

= (AB – bA) k̂

Also, N = | F | .r1

 −1 −1
|F|  A 2 + B2 
⇒ r1 =
=  
 N   (aB − bA) 
   

2 2
Ans. N = (aB – bA)k, where k is the unit vector of the z axis = |aB –bA|/ A + B

2 2
N = (aB – bA)k, tgk¡ k z v{k ds vuqfn'k ,adkd lfn'k gS = |aB –bA|/ A + B

35. A uniform cylinder of radius R and mass M can rotate freely about a stationary horizontal axis
O Fig. A thin cord of length  and mass m is wound on the cylinder in a single layer. Find the
angular acceleration of the cylinder as a function of the length x of the hanging part of the cord.
The wound part of the cord is supposed to have its centre of gravity on the cylinder axis.
From Q.8 Exercise # 2 Part - I Irodov_1.251

Rankers Varanasi office | Pandeypur Chauraha – Call 8601467389


////////////////////////

Ans : β = 2mgx/R (M + 2m)

m
Sol. λ=  


m
m1 = λx =   x


(a = α R)

m1g – T = m1a ............ (i)

 MR2 
TR=   α + (m – m1) (R2) α ............(ii)
 2
 

MR
T= α + (m – m1) R α
2

Ma
T= + (m – m1) a
2

Ma
m1g – – (m – m1) a = m1a
2

Ma
m1g – – ma + m1a = m1a
2

 Ma 
m1g =  + ma 
 2 

2m1a
α=
(M + 2m)

 mx 
2 g
   2mgx 
α=  ⇒ α = 
(M + 2m)R  (M + 2m)R 

36. A vertically oriented uniform rod of mass M and length  can rotate about a fixed horizontal
smooth axis passing through its upper end. A horizontally flying bullet of mass m strikes the
lower end of the rod and gets stuck in it; as a result, the rod swings through an angle α(α <
90º). Assuming that m << M, find :
Irodov_1.275

(a) the velocity of the flying bullet ;

Rankers Varanasi office | Pandeypur Chauraha – Call 8601467389


(b) the momentum increment in the system "bullet + rod" during the impact; what causes
the change of that momentum ;

(c) at what distance x : from the upper end of the rod the bullet must strike for the
momentum of the system "bullet-rod" to remain constant during the impact.

[Ans : (a) v = (M/m) 2 / 3g sin(α/2); (b) ∆p = M 1/ 6g sin(α/2); (c) x ≈ 2/3]

Sol. (a) About the axis of rotation of rod, the angular momentum of the system is conserved velocity
of the flying bullet is V

 M2 
mv =  m2 +  ω
 3 

mv  3mv 
ω= =   (m <<< M) ................. (i)

m +
M 
  M 
 3 

conservation of mechanical energy of the system (rod + bullet)

1  2 M2  
 m +  ω2 = (M+m)g (1– cos α ) ——(ii)
2  3  2

From (i) and (ii)

M 2g
V= sin α
m 3 2

   
(b) ∆P = m (ω ) + M  ω   – mv From v and w
  2 

1  g 
∆P = mv =  M sin α 
2 6 2 
 

 M2 
mvx =  + mx 2  ω´
 3 
 

3mvx
ω´ =
M2

final momentum

M M
pf = mx ω´ + ∫ yω´
0

dy =
2
ω´

3 x
= mv
2 

 3x   2 
∆p = pf – pi = mv  – 1 = 0 ⇒ x = 
 2   3 

Rankers Varanasi office | Pandeypur Chauraha – Call 8601467389


37. A small spherical ball of mass m is rolling without slipping down the loop track as shown in the
figure. The ball is released from rest on the linear portion at a vertical height h from the lowest
point. The circular part as shown in figure has a radius r. [g = 10 ms2]

(a) Find the kinetic energy of the ball when it is at a point A where the radius make an
angle θ with the horizontal

(b) Find the radial and the tangential accelerations of the centre when the ball isat A.

(c) Find the normal force and the frictional force acting on the ball if h = 50 cm,
r = 10 cm, θ = 0 and m = 70 g.

10  h  5
Ans. (a) mg(h-r-r sinα), (b) g  − 1 − sin α  , – g cosα (c) 4N, 0.2N upward Åij dh vksj
7 r  7

38. A point A is located on the rim of a wheel of radius R = 0.50 m which rolls without slipping
along a horizontal surface with velocity v = 1.00 m/s as shown in figure. Find:
Irodov_1.52

(a) the modulus and the direction of the acceleration vector of the point A ;

(b) the total distance s traversed by the point A between the two successive moments at which
it touches the surface.

v2
Ans : (a) ωA = = 2.0 m/s2, the vector ωA is permanently directed to the centre of the wheel ;
R

(b) s = 8R = 4.0 m

v2
Ans : (a) ωA = = 2.0 m/s2,
R

(b) s = 8R = 4.0 m

Sol.

Rankers Varanasi office | Pandeypur Chauraha – Call 8601467389


V = ωR = (For pure rolling)

(linear acceleration = 0)

rolls with out slipping so ;

acc. only centripetal acc.

v2
aA =
R

V = ωR

VA = (V – V cos θ) î + V sin θ ĵ

VA = (v – v cosθ)2 + (v sinθ)j

VA = (2V sin θ/2) θ = ωt

ds  ωt 
= 2 V sin θ\2 = 2 V sin  
dt  2 

s 2π / ω
 ωt  8v

0
ds =
O
∫ 2v sin   dt =
2
  ω
= (8R)

39. A uniform sphere of mass m and radius r rolls without sliding over a horizontal plane, rotating
about a horizontal axle OA . In the process, the centre of the sphere moves with velocity along
a circle of radius R Find the kinetic energy of the sphere. [Book_I.E. Irodov]

Ans . T = 7/10 mv2 (1 + 2/7r2/R2)]

Sol. We know K.E. from fixed axis relation is given by :

Rankers Varanasi office | Pandeypur Chauraha – Call 8601467389


1 1 1
K= Ι xx ω2x + Ι yy ω2y + Ι zz ω2z
2 2 2

v v
ωx= , ωy= & ωz= 0
r R

1 1
Then K = Ι xx ω2x + Ι yy ω2y + 0
2 2
2 2
1  2 2  v  1 2 v 
=  mr    +  mr 2 + mR2   
25  r  2 5  R 

7  2 r2 
K= mv 2  1 +
10  7 R2 
 

40. A uniform plate of mass 'm' is suspended in each of the ways shown. For each case determine
immediately after the connection at B has been released ;

(a) The angular acceleration of the plate.

(b) The acceleration of its center of mass.

Rankers Varanasi office | Pandeypur Chauraha – Call 8601467389


1.2 g
Ans. (i) (a) (cw) (b) − 0.3 ( ˆi + 2 ˆj) g

(ii) (a) 24 g/17  (cw) (b) 12 g/17↓

(iii) 2.4 g/ (cw) (b) 0.5 g↓

 C2 C2  5 M C2
Sol. (i) (a) ΙCM = M  +  =
 12 4 × 12  12 × 4

ΙA = ΙCM + Mx2

5 M C2 5 MC2 20 MC2
ΙA = + =
12 × 4 16 48

C 6 g
ΙA → mg × = ΙA α ⇒ α =
2 5 C

6 g 5C 6 g
(b) acm = α x = × = x
5 C 4 5 C

6 g C 6 g
ax = – acm cos θ = – x. = –
5 C 4 . x 20

= – 0.3 g

6 g C/2
ag = – acm sinθ = − x .
5 C x

= – 0.6 g

−0.3 g ˆi − 0.6 g ˆj
a=

(ii) (a) Mg – T = M acm .... (1)

ΙCM →

C 5 MC2 5 MC
T× = α ⇒ T= α .... (2)
2 48 24

Rankers Varanasi office | Pandeypur Chauraha – Call 8601467389


As aA = 0

(we know : acc. along the string is zero)

acm – α x cos(90 – θ) = 0

C
acm = α x sin θ = ax.
2x

α C
acm = .... (3)
2

5 MC 2 acm 5 M acm
T= = .... (4)
24 C 12

5 M acm
Mg = M acm +
12

17 M acm 12 g
= , acm = ↓
12 17

2 acm 24 g
(a) α = =
C 17C

(iii)

mg C
(a) τcm = × = Icm α
2 2

mg C 5 MC2 12 g
= α ⇒ α=
4 48 5 C

Mg
(b) FA =
2

g
Mg – FA = m acm ⇒ acm = = 0.5g ↓
2

41. Figure (1) shows a mechanical system free of any dissipation. The two spheres (A and B) are each of equal
mass m, and a uniform connecting rod AB of length 2r has mass 4m. The collar is massless. Right above
the position of sphere A in Fig. (1) is a tunnel [Olympiad_2011]

Rankers Varanasi office | Pandeypur Chauraha – Call 8601467389


from which balls each of mass m fall vertically at suitable intervals. The falling balls cause the rods and
attached spheres to rotate. Sphere B when reaches the position now occupied by sphere A, suffers a collision
from another falling ball and so on. Just before striking, the falling ball has velocity υ. All collision are elastic
and the spheres as well as the falling balls can be considered to be point masses. [Marks : 12]

(a) Find the angular velocity ωi + 1 of the assembly in terms of {ωi , υ, and r } after the ith ball has struck it.
[4]

ωi+1

(b) The rotating assembly eventually assumes constant angular speed ω*. Obtain ω* in terms of υ and r by
solving the equation obtained in part (a). Argue how a constant ω* does not violate energy conservation.
[2]

ω* =

Argument

(c) Solve the expression obtained in part (a) to obtain ωi in terms of {i,υ, and r} .

ωi =

(d) If instead of a pair of spheres, we have two pairs of spheres as shown in figure below. What would be the
new constant angular speed ω* of the assembly (i.e. the answer corresponding to part (b).
[2]

Rankers Varanasi office | Pandeypur Chauraha – Call 8601467389


ω* =

7 6 v v
Ans. (a) ωi+1 = ωi + (b) ω∗ = after this no further collision occurs
13 13 r r

v   7 
i
(c) ωi+1 = 1 −    (d) ω∗ will remain same as in case b.
r   13  
 

10 2
Sol. I ωi + mvr = Iωi +1 + m v' r I= mr
3

ω r − v' 
−  i+1
e= =1
 i −v 
ω r

ωi+1r – v' = v' – ωir

v ' = (ωi+1+ ωi) r – v

I ωi + mvr = Iωi+1 + mr ( ωi+1 + ωi ) r − v 

I ωi + mvr = Iωi+1 + mr2 ωi+1 +mr2 ωi – mvr

( I – mr2) ωi + 2mvr = (I +mr2) ωi+1

7mr 2 13mr 2
ωi + 2mvr = ωi+1
3 3

7 6
mr2 ωi+1 = mr2 ωi + mvr
13 13

7 6 v
ωi+1 = ωi +
13 13 r

v
(b) ω = after this no further collision occurs
r

6v
(c) ω2 =
13r

Rankers Varanasi office | Pandeypur Chauraha – Call 8601467389


7  6v  6v
ω3 =   +
13  13r  13r

2
 7  6v 6v 7 6v
ω4 =   13r + 13r × 13 + 13r
13
 

v   7 
i
ωi+1 = 1 −   
r   13  
 

(d) ω* will remain same as in case b.

42. A rod of length L forming an angle β with the vertical strikes a frictionless floor at A with a vertical
velocity v1 and no angular velocity. Assuming that the impact at A is perfectly elastic, derive an
expression for the angular velocity of the rod immediately after the impact. [From Q.26
Exercise # 2 Part - I ]

ν1 12 sin β
Ans. ω= (cw) (correction by SR Sir 3-9-09)
L 3 sin2 β + 1

Sol.

m
m

Coefficient of restitution

 ω 
 V2 +  2 sin β 
e=1=  
V1

ω
(V1 = V2 + sin β) ..... (i)
2

angular momentum about point A


Li = mV1 sin β
2

  
Lƒ = LCM + LA =  ICMω − mV2 sin β 
 2 

Rankers Varanasi office | Pandeypur Chauraha – Call 8601467389


Li = Lƒ

 m2 
mV1 sin β = ω – mV2 sin β ..... (ii)
2 12 2

Put equation (i) in (ii) equation

 m2  ω  
mV1 sin β = ω – m  V1 − sin β  sin β
2 12  2  2


mV1 sin β
2

m2  m2
= ω – mV1 sin β + ω sin2 β
12 2 4

 m2 m2 
 mV1 sin
= β + ω sin2 β 
12 4 
 

   V1 (12 sin β) 
V1 sin β = + ω sin2 β ω = 
12 4  3 sin2 β + 1 

43. Consider a bicyle in vertical position accelerating forward without slipping on a straight horizontal road. The
combined mass of the bicycle and the rider is M and the magnitude of the accelerating torque applied on the
rear wheel by the pedal and gear system is τ. The radius and the moment of inertia of each wheel is R and I
(with respect to axis) respectively. The accelaration due to gravity is g.
[INPhO-2013]

(a) Draw the free diagram of the system (bicycle and rider ).

(b) Obtains the accelaration a in terms of the above mentioned quantities.

a=

(c) For simplicity assume that the centre of mass of the system is at height R from the gruond and equidistant
at 2 R from the center of each of the wheels. Let µ be the coefficient of friction (both static and dynamic)
between the wheels and the ground. Consider M >> l/R2 and no slipping. Obtain the conditions for the
maximum acceleration am of the bike.

am =

(d) For µ = 1.0 calculate am.

am =

Sol. (a) Here f1 f2 are frictional forces and N1, N2 are normal reactions

Rankers Varanasi office | Pandeypur Chauraha – Call 8601467389


τ µg / 2
(b) a = R (c) a ≤ (d) am = 2g/3
MR2 + 2l (1 − µ / 4)

Sol. (a) Here f1 f2 are frictional forces and N1, N2 are normal reactions

τ µg / 2
(b) a = R (c) a ≤ (d) am = 2g
2 (1 − µ / 4)
MR + 2l

-------------------------------

Rankers Varanasi office | Pandeypur Chauraha – Call 8601467389

You might also like